Вы находитесь на странице: 1из 51

A 36-year-old G2P2 woman comes to the office to discuss discontinuing contraception.

Six weeks ago, she had her first Depo-Provera injection and now she has unpredictable bleeding. She is frustrated and irritated by these symptoms. She has a history of hypertension but is currently on no medications. Vital signs reveal a blood pressure of 130/90; weight 188 pounds, height 65 inches; BMI 31.4kg/m2. Which of the following is the most appropriate next step in the management of this patient? a) Offer a different method of contraception b) Reassurance that this is normal initially c) Begin oral contraceptives as a backup method for contraception d) Perform an endometrial biopsy to assess for endometrial cancer e) Offer add-back estrogen therapy She should be reassured since initially after Depo-Provera injection there may be unpredictable bleeding. This usually resolves in 2 or 3 months. In general, after 1 year of using Depo-Provera, nearly 50% of users have amenorrhea. b) Reassurance that this is normal initially An 18-year-old G0 woman with LMP 14 days ago presents to the office requesting contraception, as she had unprotected intercourse the night before. She has taken oral contraceptives before and wants to know if she can just start them again now. After counseling and a discussion of long-term contraception, as well as emergency contraception, you advise her to take a low dose oral contraceptive. Of the following, which is the most appropriate instruction to give her at this time? a) If pregnancy occurs, termination is indicated b) Begin taking the pills 72 hours after intercourse c) Taking 5 pink/active tablets in two doses 12 hours apart has the same efficacy as taking them at the same time d) She should not expect any changes in her next cycle since emergency contraception can be taken at any time during a womans cycle e) Insert the second dose per vagina or take an antiemetic 1-hour before dosing to decrease nausea Emergency contraceptive pills are not an abortifacient, and they have not been shown to cause any teratogenic effect if inadvertently administered during pregnancy; therefore, termination is not indicated. They are more effective the sooner they are taken after unprotected intercourse, and it is recommended that they be started within 72 hours, and no later than 120 hours. Plan B, the levonorgestrel pills can be taken in one or two doses and cause few side effects. Oral contraceptives need to be taken 12 hours apart. Emergency contraceptive pills may be used anytime during a womans cycle, but may impact the next cycle, which can be earlier or later with bleeding ranging from light, to normal, to heavy. Taking an antiemetic one-hour before the dose will decrease her nausea. Bypassing the oral route by inserting the medication per vagina will also have the same result and allow for appropriate absorption. e) Insert the second dose per vagina or take an antiemetic 1-hour before dosing to decrease nausea A 35-year-old G3P3 woman requests contraception. Her youngest child is 7 years old. Her periods have been regular since she discontinued breast-feeding 5 years ago. Her past medical history includes depression that is controlled with antidepressants, and a history of deep venous thrombosis. She denies smoking or alcohol use. In the past, oral contraceptive pills have caused her to have severe gastrointestinal upset. What in her history makes her an ideal candidate for progestin-only pills? a) Depression b) Smoking history c) Severe nausea on combined oral contraceptives d) Lactation history e) Deep venous thrombosis Ideal candidates for progestin-only pills include women who have contraindications to using combined oral contraceptives (estrogen containing.) Contraindications include a history of thromboembolic disease, women who are lactating, women over age 35 who smoke or women who develop severe nausea with combined oral contraceptive pills. Progestins should be used with caution in women with a history of depression. e) Deep venous thrombosis A 24-year-old G1P1 woman comes to the office requesting contraception. Her past medical history is unremarkable, except for a family history of endometrial cancer. She denies alcohol, smoking and recreational drug use. She is in a monogamous relationship. She wants to significantly decrease her risk of having a gynecological malignancy. Of the following, what is the best method of contraception for this patient? a) Female condoms b) Male condoms c) Copper containing intrauterine device d) Combined oral contraceptives e) Cervical cap

Oral contraceptives will decrease a womans risk of developing ovarian and endometrial cancer. The earlier, higher dose oral contraceptive pills have been linked to a slight increase in breast cancer, but not the most recent pills. Women who use oral contraceptive pills have slightly higher risk of developing cervical intraepithelial neoplasia, but their risk of developing PID, endometriosis, benign breast changes and ectopic pregnancy are reduced. Both hypertension and thromboembolic disorders can be a potential side effect from using oral contraceptive pills. Although the mechanism is unclear, an association of tubal ligation and a lower incidence of ovarian cancer in BRCA-1 positive women has been reported. Male condoms and intrauterine devices will not lower her risk of ovarian cancer. d) Combined oral contraceptives A 35-year-old G3P3 woman comes to the office to discuss tubal ligation as she desires permanent sterilization. What are the non-contraceptive health benefits of female sterilization? a) Reduced risk of endometriosis b) Reduced risk of ovarian cancer c) Protection against endometrial cancer d) Reduction in menstrual blood flow e) Reduced risk of sexually transmitted diseases Tubal ligation has not been shown to reduce the risk of endometriosis, sexually transmitted diseases or endometrial cancer, nor is there a decrease in menstrual blood flow in women who have undergone a tubal ligation. There is a slight reduction in the risk of ovarian cancer, but the mechanism is not yet fully understood. b) Reduced risk of ovarian cancer A 24-year-old G2P2 woman with a history of two prior Cesarean sections desires a tubal ligation for permanent sterilization. She has two daughters, who are 3 and 1 years old. She is very sure she does not desire any more children. She is happily married and a stay-at-home-mom. What is the strongest predictor of post-sterilization regret for this patient? a) Not working outside the home b) Parity c) Marital status d) Age e) Childrens gender Approximately 10% of women who have been sterilized regret having had the procedure with the strongest predictor of regret being undergoing the procedure at a young age. The percentage expressing regret was 20% for women less than 30-years-old at the time of sterilization. For those under age 25, the rate was as high as 40%. The regret rate was also high for women who were not married at the time of their tubal ligation, when tubal ligation was performed less than a year after delivery, and if there was conflict between the women and their partners. d) Age A 32-year-old G3P3 woman comes to the office to discuss permanent sterilization. She has a history of hypertension and asthma (on corticosteroids). She has been married for 10 years. Her blood pressure is 140/94; weight 280 pounds; height 69 inches; and BMI 41.4kg/m2. You discuss with her risks and benefits of contraception. Which of the following would be the best form of permanent sterilization to recommend for this patient? a) Laparoscopic bilateral tubal ligation b) Mini laparotomy tubal ligation c) Exploratory laparotomy with bilateral salpingectomy d) Total abdominal hysterectomy e) Vasectomy for her husband Both vasectomy and tubal ligation are 99.8% effective. Vasectomies are performed as an outpatient procedure under local anesthetic, while tubal ligations are typically performed in the operating room under regional or general anesthesia; therefore carrying slightly more risk to the woman, assuming both are healthy. She is morbidly obese, so the risk of anesthesia and surgery are increased. In addition, she has chronic medical problems that make her at increase risk for surgery. e) Vasectomy for her husband A 35-year-old G3P3 woman comes to the office because she desires contraception. Her past medical history is significant for Wilsons disease, chronic hypertension and anemia secondary to menorrhagia. She is currently on no medications. Her vital signs reveal a blood pressure of 144/96. Of the following, what is the ideal contraceptive for her? a) Progestin-only pill b) Low dose combination contraceptive

c) Continuous oral contraceptive d) Copper containing intrauterine device e) Levonorgestrel intrauterine device The levonorgestrel intrauterine device has lower failure rates within the first year of use than does the copper containing intrauterine device. It causes more disruption in menstrual bleeding, especially during the first few months of use, although the overall volume of bleeding is decreased long-term and many women become amenorrheic. The levonorgestrel intrauterine device is protective against endometrial cancer due to release of progestin in the endometrial cavity. She is not a candidate for oral contraceptive pills because of her poorly controlled chronic hypertension. The progestin only pills have a much higher failure rate than the progestin intrauterine device. She is not a candidate for the copper-containing intrauterine device because of her history of Wilsons disease. e) Levonorgestrel intrauterine device A 23-year-old G0 woman comes to the office to discuss contraception. Her past medical history is remarkable for hypothyroidism and mild hypertension. She has a history of slightly irregular menses. Her best friend recently got a patch, so she is also interested in using a transdermal system (patch.) Her vital signs are: blood pressure 130/84; weight 210 pounds. What is the most compelling reason for her to use a different method of contraception? a) Age b) Hypothyroidism c) Weight d) Unpredictable periods e) Hypertension The patch has comparable efficiency to the pill in comparative clinical trials, although it has more consistent use. It has a significantly higher failure rate when used in women who weigh more than 198 pounds. The patch (Ortho-Vera) is a transdermal system that is placed on a womans upper arm or torso (except breasts). The patch slowly releases Ethinyl Estradiol and Norelgestromin, which establishes steady serum levels for 7 days. A woman should apply one patch in a different area each week for 3 weeks, then have a patch-free week, during which time she will have a withdrawal bleed. Patients should be informed about the risks of using a transdermal delivery system including higher estrogen levels and increased risk of thromboembolic events. c) Weight A 37-year-old G3P3 woman presents for a health maintenance exam. She is healthy with no history of medical problems. She had a tubal ligation 2 years ago. She had used combined oral contraceptives previously for a total of 10 years. Her husband has a strong family history of cardiac disease and her grandmother passed away at age 87 from a stroke. She is worried about having a heart attack herself. She has smoked one pack of cigarettes a day for the last 15 years and drinks alcohol 3 times a week. Which of the following is her strongest risk factor for cardiac disease? a) Past use of oral contraceptives b) Alcohol consumption c) Maternal family history d) Paternal family history e) Smoking history Correct!!! Although oral contraceptives are contraindicated in women with coronary vascular disease, past use of the pills does not increase current risk. This womans strongest risk is her smoking. Her husbands family history does not place her at risk. Her grandmothers history can potentially place her at increased risk but her own smoking is her biggest risk factor. Mild red wine consumption can potentially decrease her risk, and other alcohol consumption does not pose a significant increased risk. e) Smoking history would be awarded 10 points A 23-year-old woman with 6 weeks amenorrhea presents with lower abdominal pain and vaginal bleeding. Her temperature is 39C and the cervix is 1 cm dilated. Uterus is 8-week size and tender. There are no adnexal masses. Urine pregnancy test is positive. What is the most likely diagnosis? a) Threatened abortion b) Missed abortion c) Normal pregnancy d) Septic abortion e) Ectopic Pregnancy The patient has a septic abortion. She has fever, bleeding, cervix is open and exam findings consistent with septic abortion. Threatened abortions clinically have vaginal bleeding, positive pregnancy test and a cervical os closed or uneffaced, while missed abortions have retention of a nonviable intrauterine pregnancy for an extended period of time

(i.e. dead fetus or blighted ovum.) A normal pregnancy would have a closed cervix. Ectopic pregnancy would likely present with bleeding, abdominal pain and possibly have an adnexal mass; the cervix would typically be closed. d) Septic abortion A 23-year-old woman with 6 weeks amenorrhea presents with lower abdominal pain and vaginal bleeding. Her temperature is 39C and the cervix is 1cm dilated. Uterus is 8-week size, tender and there are no adnexal masses. Urine pregnancy test is positive. Which of the following is the most appropriate next step in the management of this patient? a) Single-agent antibiotics b) Observation c) Laparoscopy plus antibiotics d) Uterine evacuation plus antibiotics e) Medical termination of pregnancy plus antibiotics The management of septic abortion is broad spectrum antibiotics and uterine evacuation. Single agent antimicrobials do not provide coverage for the array of organisms that may be involved and therefore are not indicated. A laparoscopy can be indicated if ectopic pregnancy is suspected, but it is unlikely in this case. Medical termination is not the best option since prompt evacuation of the uterus is indicated in this case. d) Uterine evacuation plus antibiotics A 29-year-old G3P0 woman presents for evaluation and treatment of pregnancy loss. Her past medical history is remarkable for 3 early (<14 weeks gestation) pregnancy losses and a deep vein thrombosis 2 years ago. Parental karyotype was normal. Which of the following is the most appropriate next step in the management of this patient? a) Place a prophylactic cerclage b) Obtain cervical length c) Refer for in vitro fertilization d) Obtain a semen analysis e) Check antiphospholipid antibodies Antiphosphospholipid antibodies are associated with recurrent pregnancy loss. The workup for antiphospholipid syndrome includes assessment of anticardiolipin antibody status, PTT, and Russell viper venom time. There are multiple etiologies for recurrent pregnancy loss defined as >2 consecutive or >3 spontaneous losses before 20 weeks gestation. Etiologies include anatomic causes (uterine abnormalities either acquired or inherited,) endocrine abnormalities such as hyper or hypothyroidism and luteal phase deficiency, parental chromosomal anomalies, immune factors such as lupus anticoagulant and idiopathic factors. Cervical incompetence is diagnosed by history, physical exam and other diagnostic tests, such as ultrasound. The treatment is placement of a cerclage. Semen analysis would not be useful in this workup. e) Check antiphospholipid antibodies A 29-year-old G3P0 woman presents for evaluation and treatment of pregnancy loss. Her past medical history is remarkable for 3 early (<16 weeks gestation) pregnancy losses and a deep vein thrombosis 2 years ago. Her work up includes: MRI of pelvis normal, prolonged dilute Russell viper venom test and elevated anticardiolipin antibodies, normal thyroid function and prolactin. She wishes to get pregnant soon. In addition to aspirin, which of the following treatments is appropriate for this patient? a) No additional treatment b) Corticosteroid c) Heparin d) Levothroxine e) Bromocriptine The prolonged dilute Russell viper venom time leads one to suspect that the etiology of recurrent pregnancy loss is due to antiphospholipid antibody syndrome. The treatment is aspirin plus heparin. There is roughly a 75% success rate with combinations therapy versus aspirin alone. There is conflicting evidence regarding steroid use for treatment and heparin is more effective. c) Heparin A 25-year-old G1P0 woman at 6 weeks gestational age comes to the office because of undesired pregnancy. You discuss with her the risks and benefits of surgical versus medical abortion using misoprostol and mifepristone. Compared to surgical abortion, which of the following is increased in a woman undergoing a medical abortion? a) Post abortion pain b) Lower failure rate

c) Long-term psychologic sequelae d) Blood loss e) Future fertility In early in pregnancy (less than 49 days) both medical and surgical procedures can be offered. Mifepristone (an antiprogestin) can be administered, followed by misoprostol (a prostaglandin) to induce uterine contractions to expel the products of conception. This approach has proven to be effective (96%) and safe. A surgical termination is required in the event of failure or excessive blood loss. It seems to be more desirable by some patients as they do not have to undergo a surgical procedure; however, it is associated with higher blood loss than surgical abortion. It does not affect future fertility. Any termination of pregnancy, whether medical or surgical, can have psychological sequelae. d) Blood loss A 36-year-old G2P0 woman at 11 weeks gestational age requests a surgical termination of pregnancy. She had a manual vacuum aspiration last year and would like to undergo the same procedure again. She has chronic hypertension and diabetes well controlled on medications. Vital signs reveal a blood pressure of 120/80 and fasting blood glucose of 100. Which of the following is a contraindication for manual vacuum aspiration of this patient? a) Age b) Parity c) Gestational age d) Chronic hypertension e) Diabetes Manual vacuum aspiration is more than 99% effective in early pregnancy (less than 8 weeks.) Age, parity and medical illnesses are not contraindications for manual vacuum aspiration. Although the risk of Ashermans syndrome increases with each subsequent pregnancy termination, this patient may still undergo surgical termination as long as she understands risks and benefits. Complications of pregnancy termination increase with increasing gestational age. c) Gestational age A 25-year-old G1 woman at 20 weeks gestation desires termination of the pregnancy. Her prenatal course has been unremarkable except for a chromosomal analysis positive for Trisomy 18. She desires an autopsy of the fetus. Which of the following is the most appropriate next step in the management of this patient? a) Perform a dilatation and curettage b) Perform a dilatation and evacuation c) Await fetal demise then start induction of labor d) Perform an induction with intraamniotic hypertonic saline e) Perform an induction with intravaginal prostaglandins Both medical and surgical abortions are options for this patient, depending on her personal preferences. However, if she desires an autopsy, she must undergo a medical abortion in order to have an intact fetus. Abortion is legal until viability is achieved (24 weeks gestation) unless a fetal anomaly inconsistent with extrauterine life is identified. A dilatation and curettage is performed if the fetus is less than 16 weeks, while dilatation and evacuation can be performed after 16 weeks by those trained in the procedure. Inductions with hypertonic saline have a high morbidity, so are no longer performed. e) Perform an induction with intravaginal prostaglandins A 23-year-old G1P0 woman at 6 weeks gestation undergoes a medical termination of pregnancy. One day later, she presents to the emergency room with bleeding and soaking more than a pad per hour for the last 5 hours. Her blood pressure on arrival is 110/60; heart rate 86. On exam, her cervix is 1cm dilated with active bleeding. Hematocrit on arrival is 29%. Which of the following is the most appropriate next step in the management of this patient? a) Admit for observation b) Repeat hematocrit in 6 hours c) Begin transfusion with O-negative blood d) Give an additional dose of prostaglandins e) Prepare her for a dilatation and curettage This patient is having heavy bleeding as a complication of medical termination of pregnancy. This is managed best by performing a dilatation and curettage. It is not appropriate to wait 6 hours before making a decision regarding next step in management, or to just admit her for observation. Since the patient is not symptomatic from her anemia, it is not necessary to transfuse her at this time. e) Prepare her for a dilatation and curettage

A 22-year-old G1P0 woman with LMP 6 weeks ago presents for elective termination of her pregnancy. She is healthy with no medical problems. Her pregnancy thus far has been uncomplicated. An ultrasound performed in the office shows an 8mm endometrial stripe with no intrauterine gestational sac and no adnexal masses. Which of the following is the most appropriate next step in the management of this patient? a) Obtain a beta-hCG level b) Perform dilation and curettage c) Give patient methotrexate d) Perform a diagnostic laparoscopy e) Repeat ultrasound in 2 weeks Even though the patient reports being pregnant, she is asymptomatic with no gestational sac in the uterus. First step in her management is to establish pregnancy by obtaining a beta-hCG level. One should not assume she has an intrauterine pregnancy and perform a D&C or assume that she has an ectopic pregnancy and treat her with methotrexate or surgery until the pregnancy is confirmed. a) Obtain a beta-hCG level A 24-year-old G2P1 woman who underwent an elective termination two days ago presents to the emergency room with abdominal and pelvic pain. She has been feeling nauseated and reports a fever at home to 101.3F. On presentation, her blood pressure is 100/60; pulse 100; respiration 16; temperature 102F. Physical examination reveals diffuse abdominal tenderness and, on pelvic exam, she has marked cervical motion tenderness. In addition to sending a CBC and cultures, which of the following is the most appropriate next step in the management of this patient? a) Obtain beta-hCG level b) Proceed with a laparoscopy c) Proceed with a dilatation and curettage d) Begin IV antibiotics e) Order a hysterosonogram This patient has postoperative endometritis that could be due to introduction of bacteria into the uterine cavity at the time of dilatation and curettage. It is important to begin antibiotics immediately. After starting antibiotics, an ultrasound should be obtained to look for products of conception. If found, then the patient would require a repeat dilatation and curettage. An hCG level would not be helpful 2 days after the termination. Hysterosonogram is contraindicated when infection is present. There are no indications for laparoscopy in this patient. d) Begin IV antibiotics

A 32-year-old P0 woman, with a last menstrual period three weeks ago, presents with a 3-month history of a malodorous vaginal discharge. She reports no pruritus or irritation. She has been sexually active with a new partner for the last 4 months. Her past medical history is unremarkable. Pelvic examination reveals normal external genitalia without rash, ulcerations or lesions. Some discharge is noted on the perineum. The vagina reveals only a thin, gray homogeneous discharge. The vaginal pH is greater than 4.5. Which medication is most likely to treat her symptoms and the accompanying saline wet prep?

a) Ceftriaxone b) Doxycycline c) Metronidazole d) Azithromycin e) Penicillin Bacterial vaginosis is the most common cause of vaginitis. The infection arises from a shift in the vaginal flora from hydrogen peroxide-producing lactobacilli to non-hydrogen peroxide-producing lactobacilli, which allows proliferation of

anaerobic bacteria. The majority of women are asymptomatic; however, patients may experience a thin, gray discharge with a characteristic fishy odor that is often worse following menses and intercourse. Modified Amsel criteria for diagnosis include three out of four of the following: 1) thin, gray homogenous vaginal discharge; 2) positive whiff test (addition of potassium hydroxide releases characteristic amine odor;) 3) presence of clue cells on saline microscopy; and 4) elevated vaginal pH >4.5. Treatment consists of Metronidazole 500mg orally BID for 7 days, or vaginal Metronidazole 0.75% gel 5-grams QHS for 5 days. c) Metronidazole A 64-year-old G2P2 woman presents with a 12-month history of severe vulvar pruritus. She has applied multiple over-the-counter topical therapies without improvement. She has no significant vaginal discharge. She is currently on oral estrogen therapy for osteoporosis. She has severe introital dyspareunia and has stopped having intercourse because of the pain. Her past medical history is significant for allergic rhinitis and hypertension. On pelvic examination, the external genitalia show loss of the labia minora with resorption of the clitoris (phimosis). The vulvar skin appears thin and pale and involves the perianal area. No ulcerations are present. The vagina is mildly atrophic, but appears uninvolved. Which of the following is the most likely diagnosis in this patient? a) Squamous cell hyperplasia b) Lichen sclerosus c) Lichen planus d) Candidiasis e) Vulvar cancer

Lichen sclerosus is a chronic inflammatory skin condition that most commonly affects Caucasian premenarchal girls and postmenopausal women. The exact etiology is unknown, but is most likely multifactorial. Patients typically present with extreme vulvar pruritus and may also present with vulvar burning, pain and introital dyspareunia. Early skin changes include polygonal ivory papules involving the vulva and perianal areas, waxy sheen on the labia minora and clitoris, and hypopigmentation. The vagina is not involved. More advanced skin changes may include fissures and erosions due to a chronic itch-scratch-itch cycle, mucosal edema and surface vascular changes and, ultimately, scarring with loss of normal architecture, such as introital stenosis, and resorption of the clitoris (phimosis) and labia minora. Treatment involves use of high-potency topical steroids. There is less than a 5% risk of developing squamous cell cancer within a field of lichen sclerosus. b) Lichen sclerosus would be awarded 10 points A 22-year-old P0 woman, presents with a one-month history of profuse vaginal discharge with mild odor. She has a new sexual partner with whom she has had unprotected intercourse. She reports mild to moderate irritation, pruritus and pain. She thought she had a yeast infection, but had no improvement after using an over-the-counter antifungal cream. She is concerned about sexually transmitted infections. Her medical history is significant for lupus and chronic steroid use. Pelvic examination shows normal external genitalia, an erythematous vagina with a copious, frothy yellow discharge and multiple petechiae on the cervix. Vaginal pH is 7. Saline wet mount reveals motile, flagellated organisms and multiple white blood cells. Which of the following is the most appropriate treatment for this patient? a) Clindamycin b) Azithromycin c) Metronidazole d) Ampicillin e) Doxycycline This patient has signs and symptoms of trichomoniasis, which is caused by the protozoan, T. vaginalis. Many infected women have symptoms characterized by a diffuse, malodorous, yellow-green discharge with vulvar irritation. However, some women have minimal or no symptoms. Diagnosis of vaginal trichomoniasis is performed by saline microscopy of vaginal secretions, but this method has a sensitivity of only 60% to 70%. The CDC recommended treatment is metronidazole 2 grams orally in a single dose. An alternate regimen is metronidazole 500mg orally twice daily for seven days. The patients sexual partner also should undergo treatment prior to resuming sexual relations. c) Metronidazole A 42-year-old G2P2 woman presents with a 2-week history of a thick, curdish white vaginal discharge and pruritus. She has not tried any over-the-counter medications. She is currently single and not sexually active. Her medical history is remarkable for recent antibiotic use for bronchitis. On pelvic examination, the external genitalia show marked erythema with satellite lesions. The vagina appears erythematous and edematous with a thick white discharge. Saline wet prep reveals multiple white blood cells, but no clue cells or trichomonads.

Potassium hydroxide prep shows the following organisms, vaginal pH is 4.0. The cervix appears normal and the remainder of the exam is unremarkable except for mild vaginal wall tenderness. Which of the following is the most appropriate treatment for this patient? a) Clindamycin b) Azole cream c) Metronidazole d) Doxycycline e) Ciprofloxacin

Vulvovaginal candidiasis (VVC) usually is caused by C. albicans, but is occasionally caused by other Candida species or yeasts. Typical symptoms include pruritus and vaginal discharge. Other symptoms include vaginal soreness, vulvar burning, dyspareunia and external dysuria. None of these symptoms are specific for VVC. The diagnosis is suggested clinically by vulvovaginal pruritus and erythema with or without associated vaginal discharge. The diagnosis can be made in a woman who has signs and symptoms of vaginitis when either: a) a wet preparation (saline or 10% KOH) or Gram stain of vaginal discharge demonstrates yeasts or pseudohyphae; or b) a vaginal culture or other test yields a positive result for a yeast species. Treatment for uncomplicated VVC consists of short-course topical Azole formulations (1-3 days,) which results in relief of symptoms and negative cultures in 80%-90% of patients who complete therapy. b) Azole cream A 52-year-old nulliparous woman presents with long-standing vulvar and vaginal pain and burning. She has been unable to tolerate intercourse with her husband because of introital pain. She had difficulty sitting for prolonged periods of time or wearing restrictive clothing because of worsening vulvar pain. She recently noticed that her gums bleed more frequently. She avoids any topical over-the-counter therapies because they intensify her pain. Her physical examination is remarkable for inflamed gingiva and a whitish reticular skin change on her buccal mucosa. A fine papular rash is present around her wrists bilaterally. Pelvic examination reveals white plaques with intervening red erosions on the labia minora as noted below. A speculum cannot be inserted into her vagina because of extensive adhesions. The cervix cannot be visualized. Which of the following is the most likely diagnosis in this patient? a) Squamous cell hyperplasia b) Lichen sclerosus c) Lichen planus d) Genital psoriasis e) Vulvar cancer

Lichen planus is a chronic dermatologic disorder involving the hair-bearing skin and scalp, nails, oral mucous membranes and vulva. This disease manifests as inflammatory mucocutaneous eruptions characterized by remissions and flares. The exact etiology is unknown, but is felt to be multifactorial. Vulvar symptoms include irritation, burning, pruritus, contact bleeding, pain and dyspareunia. Clinical findings vary with a lacy, reticulated pattern of the labia and perineum, with or without scarring and erosions as well. With progressive adhesion formation and loss of normal architecture, the vagina can become obliterated. Patients may also experience oral lesions, alopecia and extragenital rashes. Treatment is challenging, since no single agent is universally effective and consists of multiple supportive therapies and topical superpotent corticosteroids. c) Lichen planus A 27-year-old P0 woman presents with a 3-year history of dyspareunia. She reports a history of always having painful intercourse, but she is now unable to tolerate intercourse at all. She has avoided sex for the last 6 months. She describes severe pain with penile insertion. On further questioning, she reports an inability to use tampons because of painful insertion. She also notes a remote history of frequent yeast infections while she was on antibiotics for recurrent sinusitis that occurred years ago. Her medical history is unremarkable, and she is on no medications. Pelvic examination is remarkable for normal appearing external genitalia. Palpation of the vestibule with a Q-tip elicits marked tenderness and slight erythema. A normal-appearing discharge is noted. Saline wet prep shows only a few white blood cells, and potassium hydroxide testing is negative. Vaginal pH is 4.0. The cervix and uterus are unremarkable. Which of the following is the most likely diagnosis in this patient? a) Vaginal cancer b) Genital herpes infection

c) Vulvar vestibulitis d) Contact dermatitis e) Chlamydia infection Vulvar vestibulitis syndrome consists of a constellation of symptoms and findings limited to the vulvar vestibule, which include severe pain on vestibular touch or attempted vaginal entry, tenderness to pressure and erythema of various degrees. Symptoms often have an abrupt onset and are described as a sharp, burning and rawness sensation. Women may experience pain with tampon insertion, biking or wearing tight pants, and avoid intercourse because of marked introital dyspareunia. Vestibular findings include exquisite tenderness to light touch of variable intensity with or without focal or diffuse erythematous macules. Often, a primary or inciting event cannot be determined. Treatment includes use of tricyclic antidepressants to block sympathetic afferent pain loops, pelvic floor rehabilitation, biofeedback, and topical anesthetics. Surgery with vestibulectomy is recommended for patients who do not respond to standard therapies and are unable to tolerate intercourse. c) Vulvar vestibulitis A 30-year-old G1P1 woman presents with a history of chronic vulvar pruritus. The itching is so severe that she scratches constantly and is unable to sleep at night. She reports no significant vaginal discharge or dyspareunia. She does not take antibiotics. Her medical history is unremarkable. Pelvic examination reveals normal external genitalia with marked lichenification (increased skin markings) and diffuse vulvar edema and erythema. Saline microscopy is negative. Potassium hydroxide testing is negative. Vaginal pH is less than 4.5. The vaginal mucosa is normal. Which of the following is the most likely diagnosis in this

patient?

a) Lichen simplex chronicus b) Lichen sclerosus c) Lichen planus d) Candidiasis e) Vulvar cancer Lichen simplex chronicus, a common vulvar non-neoplastic disorder results from chronic scratching and rubbing, which damages the skin and leads to loss of its protective barrier. Over time, a perpetual itch-scratch-itch cycle develops, and the result is susceptibility to infection, ease of irritation and more itching. Symptoms consist of severe vulvar pruritus, which can be worse at night. Clinical findings include thick, lichenified, enlarged and rugose labia, with or without edema. The skin changes can be localized or generalized. Diagnosis is based on clinical history and findings, as well as vulvar biopsy. Treatment involves a short-course of high-potency topical corticosteroids and antihistamines to control pruritus. a) Lichen simplex chronicus A 20-year-old nulliparous woman college student presents with a one-month history of profuse vaginal discharge and mid-cycle vaginal spotting. She uses oral contraceptives and she feels her irregular bleeding is due to the pill. She is sexually active and has had a new partner within the past 3 months. She reports no fevers or lower abdominal pain. She has otherwise been healthy. On pelvic examination, a thick yellow endocervical discharge is noted. Saline microscopy reveals multiple white blood cells, but no clue cells or trichomonads. Potassium hydroxide testing is negative. Vaginal pH is 4.0. No cervical motion tenderness or uterine/adnexal tenderness is present. Testing for Gonorrhea and Chlamydia is performed, but those results will not be available for several days and the student will be leaving for Europe tomorrow. Which of the following is the most appropriate treatment for this patient? a) Metronidazole and Erythromycin b) Ceftriaxone and Azithromycin c) Ampicillin d) Doxycycline e) No treatment is necessary until all tests results are known

Mucopurulent cervicitis (MPC) is characterized by a mucopurulent exudate visible in the endocervical canal or in an endocervical swab specimen. MPC is typically asymptomatic, but some women have an abnormal discharge or abnormal vaginal bleeding. MPC can be caused by chlamydia trachomatis or Neisseria gonorrhoeae; however, in most cases neither organism can be isolated. Patients with MPC should be tested for both of these organisms. The results of sensitive tests for C. trachomatis or N. gonorrhoeae (e.g. culture or nucleic acid amplification tests) should determine the need for treatment, unless the likelihood of infection with either organism is high or the patient is unlikely to return for treatment. Antimicrobial therapy should include coverage for both organisms, such as azithromycin or doxycycline for chlamydia and a cephalosporin or quinolone for gonorrhea. Uncomplicated cervicitis, as in this patient, would require only 125 mg of Ceftriaxone in a single does. Ceftriaxone 250 mg is necessary for the treatment of upper genital tract infection, or pelvic inflammatory disease (PID). b) Ceftriaxone and Azithromycin A 37-year-old nulliparous woman, a CEO for a Fortune 500 company, presents with a one-week history of a painful vulvar ulcer. She reports no fevers, malaise or other systemic symptoms. She recently started use of a topical steroid ointment for a vulvar contact dermatitis. She is married and has no prior history of sexually transmitted infections. She reports no travel outside the United States by her husband or herself. Her last Pap smear, 6 months ago, was normal. A vulvar herpes culture later returns positive for herpes simplex virus type 2. A VDRL is nonreactive, and HIV testing is negative. Which of the following is the most likely diagnosis in this patient? a) Primary HSV episode b) Recurrent HSV-1 episode c) Recurrent HSV-2 episode d) Atypical HSV episode e) Contact dermatitis Two serotypes of HSV have been identified: HSV-1 and HSV-2. Most cases of recurrent genital herpes are caused by HSV-2. Up to 30% of first-episode cases of genital herpes are caused by HSV-1, but recurrences are much less frequent for genital HSV-1 infection than genital HSV-2 infection. Genital HSV infections are classified as initial primary, initial nonprimary, recurrent and asymptomatic. Initial, or first-episode primary genital herpes is a true primary infection (i.e. no history of previous genital herpetic lesions, seronegative for HSV antibodies.) Systemic symptoms of a primary infection include fever, headache, malaise and myalgias, and usually precede the onset of genital lesions. Vulvar lesions begin as tender grouped vesicles that progress into exquisitely tender, superficial, small ulcerations on an erythematous base. Initial, nonprimary genital herpes is the first recognized episode of genital herpes in individuals who are seropositive for HSV antibodies. Prior HSV-1 infection confers partial immunity to HSV-2 infection and thereby lessens the severity of type 2 infection. The severity and duration of symptoms are intermediate between primary and recurrent disease, with individuals experiencing less pain, fewer lesions, more rapid resolution of clinical lesions and shorter duration of viral shedding. Systemic symptoms are rare. Recurrent episodes involve reactivation of latent genital infection, most commonly with HSV-2, and are marked by episodic prodromal symptoms and outbreaks of lesions at varying intervals and of variable severity. Clinical diagnosis of genital herpes should be confirmed by viral culture, antigen detection or serologic tests. Treatment consists of antiviral therapy with Acyclovir, Famciclovir or Valacyclovir. c) Recurrent HSV-2 episode A 45-year-old G2P2 woman presents with a 2 month history of vulvar pruritus. She does not have a significant vaginal discharge. Her medical history is significant for HIV diagnosis for 2 years, and a pack of cigarettes a day smoking habit for the last 20 years. Her last Pap smear, 2 years ago, showed LGSIL (low-grade squamous intraepithelial lesion). She underwent colposcopy at that time, which was adequate and consistent with CINI/mild dysplasia, and she was subsequently lost to follow-up. Her menstrual history is unremarkable. Pelvic examination is remarkable for a 3cm non-ulcerated, hyperkeratotic lesion on the right labia minora, as well as multiple papillary growths on the posterior fourchette and perineum. The vagina and cervix are grossly normal. In addition to performing a Pap smear, which of the following procedures is most appropriate for this patient? a) Vaginal biopsy b) Vulvar biopsy c) Endometrial biopsy d) Cervical biopsy e) Colposcopy Genital condylomata, or warts, are typically caused by human papillomavirus (HPV) types 6 or 11. Other HPV types in the anogenital area (e.g. types 16, 18, 31, 33 and 35) have been strongly associated with cervical neoplasia and cancer. Vulvar intraepithelial neoplasia (VIN) is a possibility in this patient, given her history of cervical dysplasia, tobacco use and HIV status. Genital condylomata that do not respond to topical therapies should be biopsied.

Likewise, vulvar biopsy is indicated to evaluate the hyperkeratotic lesion in this patient and rule out the possibility of vulvar neoplasia. b) Vulvar biopsy A 26-year-old G2P2 woman reports that she is sexually active with a new male partner. She is using oral contraception for birth control and, as such, did not use a condom. She reports the new onset of vulvar burning and irritation. She thought she had a cold or the flu about 10 days ago. Given her history, which of the following is the most likely diagnosis in this patient? a) Herpes simplex virus b) Human papilloma virus c) Human immunodeficiency virus d) Molluscum contagiosum e) Trichomonas The patient is most likely infected with herpes. Herpes simplex virus is a highly contagious DNA virus. Initial infection is characterized by viral like symptoms preceding the appearance of vesicular genital lesions. A prodrome of burning or irritation may occur before the lesions appear. With primary infection, dysuria due to vulvar lesions can cause significant urinary retention requiring catheter drainage. Pain can be a very significant finding as well. Treatment is centered on care of the local lesions and the symptoms. Sitz baths, perineal care and topical Xylocaine jellies or creams may be helpful. Anti-viral medications, such as Acyclovir, can decrease viral shedding and shorten the course of the outbreak somewhat. These medications can be administered topically or orally. Human papilloma virus is the causative agent for condylomata and cervical dysplasia. It is often asymptomatic, unless vulvar lesions are present causing vulvar itching. In general, human papilloma virus is not associated with generalized symptoms like malaise. Human immunodeficiency virus is an RNA retrovirus transmitted via sexual contact or sharing intravenous needles. Vulvar burning, irritation or lesions are not typically noted with this disease, although generalized malaise can be. HIV can present with many different signs and symptoms therefore risk factors should be considered, and testing offered. Molluscum contagiosum which is caused by Poxviridae is a benign skin disease which presents as a small nodule or dome on the skin with an umbilicated center. It can be transmitted both sexually and from casual contact. Treatment involves local curettage of the lesion or cryotherapy. Trichomonas is a protozoan and is transmitted via sexual contact. It typically presents with a non-specific vaginal discharge. It does not have a systemic manifestation. a) Herpes simplex virus A 32-year-old G3P1 woman presents to your office for an annual examination. During the course of your discussion, the patient mentions that she has had an annoying vulvar irritation and slight itch for months. She sometimes notices an odor, but does not experience a specific discharge. She is sexually active with the same male partner for the past 6 months and they occasionally use condoms. On examination, she has a grayish, frothy discharge within the vaginal vault. You perform a wet prep of her vaginal secretions which shows flagellated organisms. Which of the following is the most likely diagnosis in this patient? a) Yeast, Non-Candida albicans b) Trichomonas c) Bacterial vaginosis d) Recent intercourse e) Chlamydia Trichomonas is a flagellated protozoan that can be seen on microscopy. They are slightly larger in size than a polymorphonucleated white blood cell. Pseudohyphae are seen when candida species are present. They are most easily seen after the application of KOH to a wet smear of vaginal secretions. Clue cells are vaginal epithelial cells that appear stippled with adherent bacteria. They are visible on microscopy when vaginal secretions are suspended in saline. They are seen in bacterial vaginosis. Chlamydia and gonorrhea cannot be identified on a wet prep. b) Trichomonas A 26-year-old G1P0 woman presents to your office for her first prenatal visit. As part of the routine prenatal evaluation, you offer her testing for sexually transmitted infections. Which of the following sexually transmitted infections can only be diagnosed using a blood sample? a) Chlamydia b) Human papilloma virus c) Hepatitis B d) Syphilis e) Herpes Hepatitis B screening, done through a blood sample, detects the outer shell of the Dane particle of the virus (HBsAg). Patients who have been immunized should have detectable antibody to Hepatitis B, but will NOT have antigen present. Chlamydia can be detected from urine or endocervical swab. Gonorrhea can also be detected by obtaining an

endocervical swab, and gonorrhea and chlamydia should always be tested as a pair, since the presence of one often means the presence of the other. Human papilloma virus is associated with cervical dysplasia as well as genital warts. The presence of HPV can be detected either through an abnormal Pap smear, by the visualization of condylomatous lesions, by biopsy or by HPV DNA hybridization. The spirochetes of syphilis, obtained from ulcerated lesions, can be visualized under darkfield microscopy during primary and secondary infection. In the latent phase, serologic testing is required and, for screening purposes, either an RPR (rapid plasma regain) or VDRL (venereal disease reference laboratory) test can be ordered. Herpes virus can be diagnosed by culture, as well as antibodies in the blood. c) Hepatitis B A 17-year-old G0 sexually active woman presents to the emergency room with pelvic pain that began within the last day. She reports menarche at the age of 15 and coitarche soon thereafter. She has had 4 male partners, including her new boyfriend of a few weeks. Her blood pressure is 100/60, pulse 100, and temperature 102.5F. On speculum examination, you note a foul-smelling mucopurulent discharge from her cervical os and she has significant tenderness with manipulation of her uterus. What is the next best step in the management of this patient? a) Outpatient treatment with oral broad spectrum antibiotics b) Outpatient treatment with intramuscular and oral broad spectrum antibiotics c) Inpatient treatment, intravenous antibiotics, and dilatation and curettage d) Inpatient treatment, laparoscopy with pelvic lavage e) Inpatient treatment and intravenous antibiotics The most likely cause of the symptoms and signs in this patient is infection with a sexually transmitted disease. The most likely organisms are both N. gonorrhoeae and chlamydia, and the patient should be treated empirically for both after appropriate cervical cultures are obtained. Since the patient also has a high fever, inpatient admission is recommended for aggressive intravenous antibiotic therapy in an effort to prevent scarring of her fallopian tubes and possible future infertility. e) Inpatient treatment and intravenous antibiotics A 36-year-old G0 woman presents to the emergency department accompanied by her female partner. The patient is febrile and notes severe belly pain. She states that this pain began 2-3 days ago and was associated with diarrhea as well as some nausea. It has gotten progressively worse and she has now developed a fever. Neither her partner, nor other close contacts report any type of viral illness. She had her appendix removed as a teenager. On examination, her temperature is 38.5C, her abdomen is tender with mild guarding and rebound, and she has an elevated white count. On pelvic examination, she is exquisitely tender, such that you cannot complete your exam. Pelvic ultrasound demonstrates bilateral 3-4 cm complex masses. What is the most likely underlying pathogenesis of her illness? a) Diverticulitis b) A sexually transmitted infection c) Reactivation of an old infection d) Ascending infection e) Pyelonephritis Although salpingitis is most often caused by sexually transmitted agents, such as gonorrhea and chlamydia, any ascending infection from the genitourinary tract or gastrointestinal tract can be causative. The infection is polymicrobial consisting of aerobic and anaerobic organisms such as E. coli, Klebsiella, G. vaginalis, Prevotella, Group B streptococcus and/or enterococcus. Although diverticulitis is a part of the differential diagnosis initially, the specific findings on examination and ultrasound are more suggestive of bilateral tubo-ovarian abscesses. Even though this patient does not have the typical risk factors for salpingitis, the diagnosis should be considered and explained to the patient in a sensitive and respectful manner. The patient should also be questioned separate from her partner regarding the possibility of other sexual contacts. d) Ascending infection A 16-year-old G0 woman presents to the emergency department with a two day history of belly pain. She is sexually active with a new partner and is not using any form of contraception. Temperature is 100.5F. On exam, she has lower abdominal tenderness and guarding. On pelvic exam, she has diffuse tenderness over the uterus and bilaterally. Beta hCG <5. What is the most likely diagnosis in this patient? a) Ectopic pregnancy b) Appendicitis c) Acute Cystitis d) Intrauterine pregnancy e) Acute salpingitis The signs and symptoms of acute salpingitis can vary and be very subtle with mild pain and tenderness, or the patient

can present in much more dramatic fashion with high fever, mucopurulent cervical discharge and severe pain. Important diagnostic criteria include lower abdominal tenderness, uterine/adnexal tenderness and mucopurulent cervicitis. e) Acute salpingitis A 16-year-old G0 woman presents to the emergency department with a two-day history of belly pain and nausea. She is sexually active with a new partner and is not using any form of contraception. On examination, her temperature is 37.9C, and she has bilateral lower quadrant pain, with slight rebound and guarding. On pelvic examination, she has purulent cervical discharge and cervical motion tenderness. Her white count is elevated. What is the most appropriate next step in the management of this patient? a) Methotrexate b) Oral antibiotics c) Diagnostic laparoscopy d) IV antibiotics e) No treatment until culture results are back Although some patients can be treated with an outpatient regimen, this patient should be hospitalized for IV treatment, as she is young, nulliparous, and has nausea so she might not be able to tolerate oral medications. She is also at risk for non-compliance with an outpatient treatment regimen. It is important to treat aggressively in order to prevent the long-term sequelae of acute salpingitis. Her recent sexual contacts should also be informed (by her and/or with her consent) and treated. According to the 2006 CDC recommendations, there are two options for parenteral antibiotics, making sure that both gonorrhea and chlamydia are covered. Cefotetan PLUS Doxycycline or Clindamycin PLUS Gentamicin. For outpatient treatment, the 2006 CDC guidelines recommend the following: Cefoxitin or Ceftriaxone and Probenecid 1 gram PLUS Doxycycline; or Ofloxacin with Metronidazole. d) IV antibiotics A 32-year-old G0 woman comes to your office because she has been unable to conceive for one year. She has regular menses every 27-34 days. She is currently in a mutually monogamous relationship, reports menarche at the age of 13, with coitarche at the age of 15; she has had 7 sexual partners, including her husband of 3 years. The patient reports that she has had an abnormal Pap smear about 4 years ago, but things have been normal since. She also states that, in her early twenties, she had problems with female infections and was on lots of antibiotics, but was never hospitalized. She denies any pain with intercourse and she denies any intermenstrual bleeding. Her husband has fathered 2 children from a previous marriage. What is the most likely underlying cause of infertility in this patient? a) Male factor infertility b) Cervical stenosis c) Ovulatory dysfunction d) Tubal disease e) Endometriosis Salpingitis can develop in 15-30% of women with inadequately treated gonococcal or chlamydial infections. Unrecognized salpingitis can result in significant long-term sequelae, such as chronic pelvic pain, hydrosalpinx, tubal scarring and ectopic pregnancy. Given this patients history, her inability to conceive is most likely due to the long-term sequelae of a sexually transmitted infection. Her husband has previously fathered two children, thus a varicocele or azoospermia are unlikely. The patient has no risk factors for cervical stenosis. She is having regular cycles; therefore, anovulation and pituitary dysfunction are also less likely. This case emphasizes the importance of aggressive screening and treatment protocols for sexually transmitted infections, as well as counseling regarding abstinence and safer sex practices. d) Tubal disease A 20-year-old G1P0 woman comes in for an annual gynecologic examination and for a refill on her birth control pills. She has heard something about a new vaccine to prevent a pelvic infection or something. She wants to get more information about preventing sexually transmitted infections, and know if she can get the new vaccine. Against which disease should she be vaccinated? a) Herpes Simplex Virus-1 (HSV-1) b) Herpes Simplex Virus-2 (HSV-2) c) Hepatitis A d) Hepatitis C e) HPV (Human Papilloma Virus) STIs have a very high prevalence within the population of the United States. HPV is estimated to infect 20 million Americans. There are 3 million new cases of chlamydia every year, and gonorrhea is newly diagnosed 600,000 times a year. Over 50 million adults are infected with herpes simplex virus, with 1.5 million new cases each year. Given that

most men and women with STIs are asymptomatic, the infections are highly communicable and can result in significant long-term sequelae, such as infertility, ectopic pregnancy, liver failure or death, aggressive prevention, screening and treatment is recommended. Of the above STIs, only HPV has a currently approved vaccination that results in prevention of cervical precancerous and noninvasive cervical cancers associated with HPV Types 16 and 18, the HPV viral types that account for 70% of cervical cancers. e) HPV (Human Papilloma Virus) A 19-year-old G0 woman presents to the office with a two-week history of low pelvic pain and cramping. She has a new sexual partner and is on oral contraception, but also uses condoms. She is one week into her cycle. She has noted no vaginal discharge, itch or odor. She denies fevers or chills. She does note that she is on a new diet and has started drinking lots of water. As such, she notes that she is urinating much more frequently. Her exam is entirely unremarkable. Which of the following is the most appropriate next step in the management of this patient? a) A pelvic ultrasound b) Pap test c) Wet prep d) Urinalysis e) Testing for Chlamydia Mildly symptomatic or asymptomatic urinary tract infections are common in female patients. Urinary tract infection must be considered in patients who present with low pelvic pain, urinary frequency, urinary urgency, hematuria or new issues with incontinence. In addition, routine screening of pregnant patients for asymptomatic urinary tract infections at each prenatal visit is recommended in order to prevent urinary tract infection, which can cause preterm labor. A pelvic ultrasound is not indicated at this point. d) Urinalysis

A 76-year-old G3P3 woman presents to your office with worsening urinary incontinence for the past three months. She reports increase in urinary frequency, urgency and nocturia. Her exam shows mild cystocele and rectocele. A urine culture is negative. A post-void residual is 400 cc. Which of the following is the most likely diagnosis in this patient? a) Genuine stress incontinence b) Detrusor instability c) Overflow incontinence d) Functional incontinence e) Mixed incontinence Correct!!! Overflow incontinence is characterized by failure to empty the bladder adequately. This is due to an underactive detrusor muscle (neurologic disorders, diabetes or multiple sclerosis) or obstruction (postoperative or severe prolapse). A normal post-void residual (PVR) is 50-60 cc. An elevated PVR, usually >300 cc, is found in overflow incontinence. Stress incontinence occurs when the bladder pressure is greater than the intraurethral pressure. Overactive detrusor contractions can override the urethral pressure resulting in urine leakage. The mixed variety includes symptoms related to stress incontinence and urge incontinence. c) Overflow incontinence would be awarded 10 points A 76-year-old G3P3 presents to the office with worsening stress urinary incontinence for the last 3 months. She reports an increase in urinary frequency, urgency and nocturia. On exam, she has a moderate size cystocele and rectocele. A urine culture is negative. A post-void residual is 50 cc. A cystometrogram shows two bladder contractions while filling. Which of the following is the most likely diagnosis in this patient? a) Genuine stress incontinence b) Urge incontinence c) Overflow incontinence d) Functional incontinence e) Continuous incontinence Detrusor overactivity incontinence is also known as urge incontinence. Detrusor instability is due to the overactivity of the bladder muscle. Though the testing may be simple (using a Foley catheter and attached large syringe without the plunger, filling with 50-60 cc of water at a time) or complex (using computers and electronic catheters,) the uninhibited contraction of the bladder with filling makes the diagnosis. b) Urge incontinence would be awarded 10 points A 60-year-old G4P4 woman presents with a two-year history of leakage with activity such as coughing, sneezing and lifting. Her past medical history is significant for vaginal deliveries of infants over 9 pounds. She had a previous abdominal hysterectomy and bilateral salpingo-oophorectomy for uterine fibroids. She is on

vaginal estrogen for atrophic vaginitis. Physical exam shows no anterior, apical or posterior wall vaginal prolapse. Vagina is well-estrogenized. Postvoid residual was normal. Q-tip test showed a straining angle of 60 degrees from the horizon. Cough stress test showed leakage of urine synchronous with the cough. Cystometrogram revealed the absence of detrusor instability. The patient failed pelvic muscle exercises and was not interested in an incontinence pessary. Which of the following is the best surgical option for this patient? a) Retropubic urethropexy b) Needle suspension c) Anterior repair d) Urethral bulking procedure e) Colpocleisis Genuine stress incontinence (GSI) is the loss of urine due to increased abdominal pressure in the absence of a detrusor contraction. The majority of GSI is due to urethral hypermobility (straining Q-tip angle >30 degrees from horizon.) Some (<10%) of GSI is due to intrinsic sphincteric deficiency (ISD) of the urethra. Patients can have both hypermobility and ISD. Retropubic urethropexies (Burch and Marshall Marchetti) and sling procedures have the best 5year success rates for patients with GSI due to hypermobility. Needle suspensions and anterior repairs have lower 5year success rates for GSI. Urethral bulking procedures are best for patients with ISD, but with little to no mobility of the urethra. Colpocleisis is one option to treat uterine prolapse, and is not indicated for urinary incontinence. a) Retropubic urethropexy would be awarded 10 points A 70-year-old G3P3 woman presents with a 4-year history of constant leakage. Her history is significant for abdominal hysterectomy and bilateral salpingo-oophorectomy for endometriosis. She has had four anterior repairs in the past for recurrent cystocele. The leakage started six months after her last anterior repair. Pelvic exam showed no evidence of pelvic relaxation. The vagina was well-estrogenized. Q-tip test revealed a fixed, immobile urethra. Cystometrogram showed no evidence of detrusor instability. Cystourethroscopy showed no evidence of any fistula and revealed a drain pipe urethra. Which of the following is the best first treatment for this patient? a) Retropubic urethropexy b) Needle suspension c) Artificial urethral sphincter d) Urethral bulking procedure e) Sling procedure Correct!!! This is a classic example of intrinsic sphincteric deficiency. The success rates for retropubic urethropexies, needle suspension and slings are less than 50%. An obstructive or tight sling can be performed to increase the success rate, but the voiding difficulties are significant, even requiring prolonged or lifelong self-catheterization. Urethral bulking procedures are minimally invasive and have a success rate of 80% in these specific patients. Artificial sphincters should be used in patients as a last resort. d) Urethral bulking procedure would be awarded 10 points A 60-year-old woman presents with complaints of urinary frequency and urge incontinence. Past medical history is unremarkable. She is on no medications. Pelvic exam shows no evidence of pelvic relaxation. Post void residual is normal. Urine analysis is negative. A cystometrogram revealed uninhibited detrusor contractions upon filling. Which of the following is the best non-surgical treatment for this patient? a) Amitriptyline b) Oxybutynin c) Topical (vaginal) estrogen d) Pseudoephedrine e) Kegel exercises The patient has the diagnosis of detrusor instability. The parasympathetic system is involved in bladder emptying and acetylcholine is the transmitter that stimulates the bladder to contract through muscarinic receptors. Thus, anticholinergics are the mainstay of pharmacologic treatment. Oxybutynin is one example. Although the tricyclic antidepressant, amitriptyline, has anticholinergic properties, its side effects do not make it an ideal choice. Vaginal estrogen has been shown to help with urgency, but not urge incontinence. Pseudoephedrine has been shown to have alpha adrenergic properties and may improve urethral tone in the treatment of stress incontinence. Kegel exercises or pelvic muscle training are used to strengthen the pelvic floor and decrease urethral hypermobility for the treatment of stress urinary incontinence. b) Oxybutynin would be awarded 10 points A 67-year-old G3P3 woman presents to the office due to severe pelvic protrusion status post hysterectomy. She denies any incontinence. She failed conservative management with a pessary. She underwent a vaginal

surgical repair where the pubocervical fascia was plicated in the midline as well as laterally to the arcus tendineus fascia (white line.) This patient underwent the repair of what defect? a) Cystocele b) Rectocele c) Uterine prolapse d) Enterocele e) Urethral diverticulum Correct!!! Central and lateral cystoceles are repaired by fixing defects in the pubocervical fascia or reattaching it to the sidewall, if separated from the white line. Defects in the rectovaginal fascia are repaired in rectoceles. Uterine prolapse is surgically treated by a vaginal hysterectomy, but this patient already had a hysterectomy. Enteroceles are repaired by either vaginal or abdominal enterocele repairs. Vaginal vault prolapse is treated either by supporting the vaginal cuff to the uterosacral ligaments, sacrospinous ligament or sacrocolpopexy. Urethral diverticulum does not present with severe pelvic protrusion. a) Cystocele A 57-year-old G2P2 woman presents to the office with a six-month history of leaking urine, urgency, and nocturia. She describes the amount of urine loss as large and lasting for several seconds. The urine loss occurs when she is standing or sitting and is not associated with any specific activity. What is the most likely cause of this patients symptoms? a) Stress incontinence b) Overflow incontinence c) Urge incontinence d) Mixed incontinence e) Vesicovaginal fistula This patient has urge incontinence, which is caused by overactivity of the detrusor muscle resulting in uninhibited contractions, which cause an increase in the bladder pressure over urethral pressure resulting in urine leakage. Stress incontinence is caused by an increase in intra-abdominal pressure (coughing, sneezing) when the patient is in the upright position. This increase in pressure is transmitted to the bladder that then rises above the intra-urethral pressure causing urine loss. Associated structural defects are cystocele or urethrocele. Overflow incontinence is associated with symptoms of pressure, fullness, and frequency, and is usually a small amount of continuous leaking. It is not associated with any positional changes or associated events. Mixed incontinence occurs when increased intraabdominal pressure causes the urethral-vesical junction to descend causing the detrusor muscle to contract. A vesicovaginal fistula typically results in continuous loss of urine. c) Urge incontinence A 56-year-old G3P3 woman presents to the office for her annual health maintenance exam. She is in good health and is not taking any medications. She has been postmenopausal for 3 years. She had an abnormal Pap smear 10 years ago, but results have been normal every year since. She is sexually active with her husband and uses lubricant during intercourse due to mild vaginal dryness. On examination, her cervix was 1 cm above the vaginal introitus and she was noted to have a moderate bladder prolapse. Her uterus is normal in size and she has no adnexal masses, and she was non-tender. In addition to performing a Pap smear and recommending a mammogram, what is the most appropriate next step in the management of this patient? a) Cystocele repair b) Perform a pelvic ultrasound c) Total hysterectomy d) Observation e) Topical estrogen This patient is asymptomatic from her prolapse; therefore, no intervention is necessary at this point. Cystocele repairs and hysterectomies are invasive procedures which are not indicated in this asymptomatic patient. It is not necessary to obtain a pelvic ultrasound, as her uterus is normal in size and she has no adnexal masses. Topical estrogen would not help improve the prolapse, although it might help with her vaginal dryness. She seems to be doing well with the lubricants and it is not necessary to expose her to the estrogen, especially since she still has her uterus, and estrogen treatment alone may increase her risk of endometrial cancer. d) Observation A 90-year-old G7P7 woman presents with severe vaginal prolapse. The entire apex, anterior and posterior wall are prolapsed beyond the introitus. She cannot urinate without reduction of the prolapse. Hydronephrosis was noted on ultrasound of the kidneys and it is thought to be related to the prolapse. She has a long-standing history of diabetes and cardiac disease. She is not a candidate for general or regional anesthesia. She has failed a trial of pessaries. Which of the following is the next best step in the management of this patient?

a) Do nothing and observe b) Anterior and posterior repair c) Colpocleisis d) Sacrospinous fixation e) Sacrocolpopexy Because of the hydronephrosis due to obstruction, intervention is required. Anterior and posterior repairs provide no apical support of the vagina. She will be at high risk of recurrent prolapse. The sacrospinous fixation (cuff to sacrospinous-coccygeus complex) or sacrocolpopexy (cuff to sacral promontory using interposed mesh) require regional or general anesthesia. Colpocleisis is a procedure where the vagina is surgically obliterated and can be performed under local anesthesia. Recurrence is minimal. c) Colpocleisis A 65-year-old G3P3 woman presents to the office with symptoms of vaginal pressure and heaviness, which seems to worsen towards the end of the day. She has a history of three vaginal deliveries. Her surgical history is significant for history of hysterectomy for abnormal vaginal bleeding at age 45. On exam, she is found to have a large pelvic prolapse. Which of the following is the most appropriate first step for the treatment of this patients prolapse? a) Sacrospinous ligament suspension b) Transvaginal tape c) Pessary fitting d) Anterior repair e) Topical vaginal estrogen Pessary fitting is the least invasive intervention for this patients symptomatic prolapse. Although a sacrospinous ligament suspension would be an appropriate procedure for this patient, it is invasive and not an appropriate first step. Transvaginal tape is used for urinary incontinence and has no role in the management of this patient. An anterior repair can potentially help with her symptoms, depending on what is contributing most to her prolapse but, again, it is invasive. Topical estrogen is unlikely to properly treat her prolapse and related symptoms. c) Pessary fitting would be awarded 10 points A 27-year-old G0 woman presents due to a one-year history of painful periods and intercourse. Pain, when present is recently 7/10 in strength and occasionally requires that she miss work. She now tries to avoid intercourse and no longer finds it pleasurable. She is otherwise in good health. Her last menstrual period was 17 days ago and her menses are typically 28 days apart. She had chlamydia once, at age 19. Physical exam is notable for some tenderness on abdominal and pelvic exams in the lower quadrants. Uterus is normal in size, but there is some tender nodularity on the back of the uterus. What is the most likely diagnosis in this patient? a) Chronic pelvic inflammatory disease b) Endometriosis c) Adenomyosis d) Uterine retroversion e) Endometritis This patient has typical symptoms of endometriosis, including dysmenorrhea and dyspareunia. In addition, the nodularity on the back of the uterus is suggestive of endometriosis. Chronic pelvic inflammatory disease would not present this far out from a known infection. Adenomyosis is endometrial glands embedded in the wall of the uterus. Endometritis is an infection of the endometrium. The patient does not have a retroverted uterus on exam. b) Endometriosis A-29-year old G0 woman presents, due to the inability to conceive for the last year. Her cycles are regular every 28 days, but she has very painful periods, occasionally requiring that she miss work despite the use of non-steroidal anti-inflammatory medicine (NSAIDs). She also reports painful intercourse, which is becoming a problem as she now tries to avoid intercourse, even though she would like to conceive. She is otherwise in good health and has been married for 5 years. She is 5 feet 4 inches tall and weighs 130 pounds. She has a history of pelvic inflammatory disease at age 19, for which she was hospitalized. Her mother had a history of ovarian cancer at age 49. Physical exam is notable for some lower quadrant tenderness on abdominal and pelvic exams. Uterus is normal in size, but there is a slightly tender palpable left adnexal mass. A pelvic ultrasound shows a 5 cm left complex ovarian cyst and 2 simple cysts measuring 2 cm in the right ovary. What is the underlying pathophysiology of the disease process in this patient? a) Chronic pelvic inflammatory disease b) Her family history of ovarian cancer

c) Endometrial glands outside of uterine cavity d) Polycystic ovarian syndrome e) Functional hemorrhagic cysts The patient has typical signs of endometriosis, which is characterized by the presence of endometrial glands and stroma outside of the uterus. Endometriosis is present in about 30% of infertile woman. She does not have the signs and symptoms of chronic pelvic inflammatory disease. She also does not have the signs and symptoms of polycystic ovarian syndrome, which typically presents with oligomenorrhea in overweight patients. The complex ovarian cyst is most likely an endometrioma. The duration of her symptoms makes functional hemorrhagic cyst a less likely option. c) Endometrial glands outside of uterine cavity A 63-year-old G0 woman comes to the office for a health maintenance exam. She is healthy and not taking any medications. She has no history of abnormal Pap smears or sexually transmitted diseases. She has a history of endometriosis and infertility in the past. She has been postmenopausal for 10 years and is not on hormone replacement therapy. She is 5 feet 4 inches tall and weighs 130 pounds. On pelvic examination, the patient has a palpable left adnexal mass. An ultrasound was obtained, which showed a 5 cm complex left ovarian cyst. What is the most appropriate next step in the management of this patient? a) CAT scan of the abdomen and pelvis b) MRI of the pelvis c) Exploratory surgery d) Repeat ultrasound in 3 months e) Observation A complex ovarian mass in a postmenopausal patient needs to be surgically explored. Although this could be an old endometrioma which never resolved, this cannot be assumed. If this is ovarian cancer, waiting 3 months can change this patients prognosis and, most likely, it will not resolve, since this is not a physiological cyst. A CAT scan or MRI will not add more information and ultrasounds are typically the best imaging studies for the uterus and adnexa. c) Exploratory surgery A 26-year-old G0 woman returns for a follow-up visit regarding endometriosis. She has been using NSAIDs to manage her pelvic pain, but it has been getting worse recently and she has had to miss 4 days of work in the last two months. She is sexually active with her husband of 2 years, although it has been more painful recently. She has regular menstrual cycles and is using condoms for contraception. On pelvic exam, she had localized tenderness in the cul de sac and there were no palpable masses. What is the most appropriate next step in the management of this patient? a) GnRH agonist b) Observation c) Oral contraceptives d) Laparoscopy and ablation of endometriosis e) Progesterone intrauterine device Oral contraceptives will be the next best choice for this patient. They provide negative feedback to the pituitaryhypothalamic axis which, in turn, stops stimulating the ovary to produce sex hormones, such as estrogen which, in turn, stimulates endometrial tissue located outside of the endometrium and uterus. GnRH agonists also exert negative feedback, but can be used short term only and have more side effects. Observation is not the next best step, as the patient has worsening symptoms. It is appropriate for patients with limited disease and mild or no symptoms, or for those trying to conceive. Laparoscopy is indicated in the patient who fails medical treatment and/or is planning pregnancy in the near future. A progesterone intrauterine device might potentially help alleviate some of her symptoms but is not the best management for endometriosis. c) Oral contraceptives A 26-year-old G0 woman returns for a follow-up visit regarding suspected endometriosis. She has been using NSAIDs and birth control pills to help manage her pelvic pain, which has been getting worse. While discussing further treatment options, she asks if there is any test or procedure you can perform to confirm her diagnosis. Which of the following tests or procedures would you recommend? a) Pelvic ultrasound b) Blood FSH/LH ratio and estradiol level c) MRI of the pelvis d) Exploratory laparoscopy e) History and physical exam confirm the diagnosis Definitive diagnosis is based on exploratory surgery and biopsies, although endometriosis is usually initially treated based on the clinical presentation. In addition, this patient can benefit from laparoscopy, since she has failed the two most common treatments for endometriosis, NSAIDs and OCPs. There is no imaging study or blood test that can

confirm the diagnosis of endometriosis. Although often used when implementing initial treatment, history and physical exam are not sufficient to confirm the diagnosis as patients present in diverse ways. d) Exploratory laparoscopy A 48-year-old G0 woman comes to the office for a health maintenance exam. She is healthy and not taking any medications. She has no history of abnormal Pap smears or sexually transmitted infections. Her menstrual cycles are normal and her last cycle was three weeks ago. Her mother was diagnosed with endometriosis and had a hysterectomy and removal of the ovaries at age 38. She is 5 feet 4 inches tall and weighs 130 pounds. On pelvic examination, the patient had a palpable left adnexal mass. An ultrasound was obtained, which showed a 4 cm complex left ovarian cyst and a 2 cm simple cyst on the right ovary. What is the most likely diagnosis in this patient? a) An endometrioma b) A hemorrhagic cyst c) Ovarian carcinoma d) A mature teratoma e) Polycystic ovaries This patient most likely has a hemorrhagic cyst, considering her history and where she is in her menstrual cycle. Her mothers history of endometriosis does increase her risk; however, it is unlikely since she has never had any symptoms herself. Ovarian carcinoma would need to be ruled out, but it is unlikely in an otherwise asymptomatic premenopausal patient. A mature teratoma would have more pathognomonic findings on ultrasound. This patient does not have typical symptoms, body habitus or ultrasound findings for patients with polycystic ovaries. b) A hemorrhagic cyst A 48-year-old G0 woman comes to the office for a health maintenance exam. She is healthy and not taking any medications. She has no history of abnormal Pap smears or sexually transmitted diseases. She is not currently sexually active. Her menstrual cycles are normal and her last cycle was three weeks ago. She smokes one pack of cigarettes per day. Her mother was diagnosed with endometriosis and had a hysterectomy and removal of the ovaries at age 38. She is 5 feet 4 inches tall and weighs 130 pounds. On pelvic examination, the patient had a palpable left adnexal mass. An ultrasound was obtained, which showed a 4 cm complex left ovarian cyst and a 2 cm simple cyst on the right ovary. What is the most appropriate next step in the management of this patient? a) CAT scan of the abdomen and pelvis b) MRI of the pelvis c) Abdominal hysterectomy and bilateral salpingo-oophorectomy (TAH/BSO) d) Repeat ultrasound in 2 months e) Oral contraceptives A repeat ultrasound is the most appropriate next step, as this is most likely a hemorrhagic cyst which will resolve on its own. A CAT scan or MRI of the pelvis will not add any more information. There is no indication to proceed with a TAH/BSO. Oral contraceptives are contraindicated in this patient, as she is older than 35 and smokes. d) Repeat ultrasound in 2 months A 26-year-old G0 woman presents to the emergency room with severe right lower quadrant pain associated with nausea for the last 6 hours, which began shortly after she finished her aerobic exercises. She has a history of suspected endometriosis, which was diagnosed 2 years ago, based on her severe dysmenorrhea. She has been using NSAIDs with her menses to control the pain. She is not sexually active, and is otherwise in good health. Her menstrual cycles are normal every 28 days and her last menstrual period was 3 weeks ago. She has no history of sexually transmitted infections. Her blood pressure is 145/70; pulse is 100; temperature is 99.2F. She appears uncomfortable. On abdominal exam, she has moderate tenderness to palpation in the right lower quadrant. On pelvic exam, she has no lesions or discharge. A complete bimanual exam was difficult to perform due to her discomfort. BHCG <5 and WBC is upper limit normal. A pelvic ultrasound showed a 6cm right ovarian mass. The uterus and left ovary appeared normal. There was a moderate amount of free fluid in the pelvis. What is the most likely diagnosis in this patient? a) Appendicitis b) Exacerbation of the endometriosis c) Ovarian carcinoma d) Ovarian torsion e) Ovarian ectopic pregnancy The sudden onset of pain and nausea, as well as the presence of a cyst on ultrasound suggest ovarian torsion. Although appendicitis is on the differential, it is unlikely to have such a sudden onset of pain and a normal white count. Her endometriosis can get worse but it would be unlikely to be of such sudden onset. Although she has an ovarian

mass, the BHCG is negative, which rules out pregnancy. d) Ovarian torsion A 26-year-old G0 woman presents to the emergency room with 8 hours of severe right lower quadrant pain associated with nausea. She has a history of suspected endometriosis, which was diagnosed 2 years ago, based on severe dysmenorrhea. She has been using NSAIDs with her menses to control the pain. She is not sexually active. She is otherwise in good health. Her menstrual cycles are normal every 28 days and her last menstrual period was 3 weeks ago. She has no history of sexually transmitted infections. Her blood pressure is 145/70; pulse is 100; temperature is 99.2F. She appears uncomfortable. On abdominal exam, she has moderate tenderness to palpation in the right lower quadrant. On pelvic exam, she has no lesions or discharge. A complete bimanual exam was difficult to perform due to her discomfort. Labs: BHCG <5, hematocrit 29%. A pelvic ultrasound showed a 6cm right ovarian mass. The uterus and left ovary appeared normal. There was a moderate amount of free fluid in the pelvis. What is the most appropriate next step in the management of this patient? a) MRI of the pelvis b) Doppler pelvic ultrasound c) CAT scan of the pelvis d) Begin oral contraceptives e) Surgical exploration This patient most likely has ovarian torsion and needs to be surgically explored. Further imaging studies will not help beyond the information obtained on the ultrasound. A Doppler ultrasound to check the blood flow to the ovaries is controversial, as normal flow does not rule out ovarian torsion. Although oral contraceptives can help decrease the development of further cyst formation and control the pain associated with endometriosis, this patient needs immediate surgical attention due to suspected ovarian torsion. e) Surgical exploration A 29-year-old G0 woman presents due to the inability to conceive for the last two years. She has a known history of endometriosis, which was diagnosed by laparoscopy 3 years ago. She has pelvic pain, which is controlled with non-steroidal anti-inflammatory drugs. Her cycles are regular every 28 days. She is otherwise in good health and has been married for 5 years. Her husband had a semen analysis, which was normal. She had a hysterosalpingogram, which showed patent tubes bilaterally. She is getting frustrated that she has not yet gotten pregnant and asks to proceed with fertility treatments. What is the most appropriate next step in the management of this patient? a) Perform a laparoscopy b) Administer a GnRH agonist c) Ovarian stimulation with Clomiphene Citrate d) Proceed with in vitro fertilization e) Intrauterine insemination A patient with a known history of endometriosis who is unable to conceive and has an otherwise negative workup for infertility, benefits from ovarian stimulation with Clomiphene Citrate, with or without intrauterine insemination. A GnRH agonist is used to control pelvic pain in endometriosis patients unresponsive to other hormonal treatments. A laparoscopy with treatment of endometriosis can increase fertility rates but, in this patient, is better reserved if she fails medical treatment. Intrauterine insemination can be added if ovarian stimulation alone does not result in pregnancy. In vitro fertilization and adoption can be offered if other treatments fail. c) Ovarian stimulation with Clomiphene Citrate would be awarded 10 points A 17-year-old G0 woman presents with a 3-year history of severe dysmenorrhea shortly after menarche at age 14. Her menstrual cycles occur every 27-29 days and last 5-7 days, with heavy flow. She has been treated with NSAIDs and cyclic oral contraceptives for the last year without significant improvement. She misses 2-3 days of school each month due to her menses. She has never been sexually active. Physical exam is remarkable for breasts, Tanner Stage IV, and pubic hair, Tanner Stage IV. Pelvic examination is normal. Both the patient and her mother are concerned. What is the next best step in the management of this patient? a) Empiric treatment with GnRH agonist b) CT of the pelvis c) Diagnostic laparoscopy d) Pelvic ultrasound e) A hysterosalpingogram Chronic pelvic pain is the indication for at least 40% of all gynecologic laparoscopies. Endometriosis and adhesions account for more than 90% of the diagnoses in women with discernible laparoscopic abnormalities, and laparoscopy is indicated in women thought to have either of these conditions. Often, adolescents are excluded from laparoscopic evaluation on the basis of their age, but several series show that endometriosis is as common in adolescents with

chronic pelvic pain as in the general population. Therefore, laparoscopic evaluation of chronic pelvic pain in adolescents should not be deferred based on age. A GnRH agonist should not be used in a patient without confirmed diagnosis of endometriosis at such a young age. c) Diagnostic laparoscopy A 24-year-old G0 woman presents with a one-year history of introital and deep thrust dyspareunia. She also has a 2-year history of severe dysmenorrhea, despite the use of oral contraceptives. She underwent a diagnostic laparoscopy 6 months ago that showed minimal endometriosis with small implants in the posterior cul de sac only, which were ablated with a CO2 laser. On further questioning, she reports significant urinary frequency, urgency and nocturia. A recent urine culture was negative. The most likely additional diagnosis for this patient is: a) Acute cystitis b) Interstitial cystitis c) Acute urethral syndrome d) Acute urethritis e) Salpingitis Interstitial cystitis (IC) is a chronic inflammatory condition of the bladder, which is clinically characterized by recurrent irritative voiding symptoms of urgency and frequency, in the absence of objective evidence of another disease that could cause the symptoms. Pelvic pain is reported by up to 70% of women with IC and, occasionally, it is the presenting symptom or chief complaint. Women may also experience dyspareunia. The specific etiology is unknown, but IC may have an autoimmune and even hereditary component. b) Interstitial cystitis A 32-year-old G3P2, with a last menstrual period 2 weeks ago, presents with a 6-month history of abdominal pain. She has noncyclic intermittent pain, which she describes as crampy and diffuse across the lower abdomen. Her pain is typically relieved with defecation and is associated with loose, watery stools. Onset of the symptoms is associated with a change in stool frequency from once daily to multiple times daily. She also experiences bloating and abdominal distention several times a week. Her medical history is significant for chronic migraines and she denies previous surgery. Her gynecological history is unremarkable. Her abdominal and pelvic examinations are both normal, except for mild tenderness in the left lower quadrant to palpation. The most likely diagnosis in this patient is: a) Pelvic adhesions b) Diverticulosis c) Gastroenteritis d) Irritable bowel syndrome e) Mechanical gastrointestinal cause Irritable bowel syndrome (IBS) is a common functional bowel disorder of uncertain etiology. It is characterized by a chronic, relapsing pattern of abdominal and pelvic pain, and bowel dysfunction with constipation or diarrhea. IBS is one of the most common disorders associated with chronic pelvic pain. IBS appears to occur more commonly in women with chronic pelvic pain than in the general population. Diagnosis is based on the Rome II Criteria for IBS, which includes at least 12 weeks (need not be consecutive) in the preceding 12 months of abdominal discomfort or pain that has 2 of 3 features: 1) relief with defecation; 2) onset associated with a change in frequency of stool; or 3) onset associated with a change in stool form or appearance. The patients history does not support pelvic adhesions, and diverticulosis (although very common) typically may be asymptomatic unless inflammation/infection develops. In this case, the symptoms for IBS may be indistinguishable from diverticulitis or severe diverticular disease. d) Irritable bowel syndrome A 22-year-old G0 woman graduate student, presents with worsening pelvic pain. She previously underwent a laparoscopic ablation of endometriosis followed by continuous oral-contraceptive pills. She had short-term relief from this approach, but now has failed this treatment and is seeking additional medical management. Which of the following mechanisms best explains how a gonadotropin releasing hormone (GnRH) agonist would help alleviate her symptoms? a) "Down-regulation of the hypothalamic-pituitary gland production and release of LH and FSH leading to reduction in estradiol levels b) Up-regulation of the hypothalamic-pituitary gland production and release of LH and FSH leading to elevation in estradiol levels c) Suppression of both LH and FSH mid-cycle surges, resulting in the loss of ovarian production of estrogen d) Induction of a pseudopregnancy state, causing a decidual reaction in the functioning endometrial tissue e) Competitive inhibitor for estrogen receptors Gonadotropin-releasing hormone (GnRH) agonists are analogues of naturally occurring gonadotropin-releasing

hormones that down-regulate hypothalamic-pituitary gland production and the release of luteinizing hormone and follicle-stimulating hormone leading to dramatic reductions in estradiol level. The GnRH agonists available in the United States are Nafarelin, Goserelin and Leuprolide. Numerous clinical trials show GnRH agonists are more effective than placebo and as effective as Danazol in relieving endometriosis-associated pelvic pain. Danazol, a 17-alphaethinyl testosterone derivative, suppresses the mid-cycle surges of LH and FSH. Combined estrogen and progestin therapy in oral contraceptives produces the pseudopregnancy state. a) "Down-regulation of the hypothalamic-pituitary gland production and release of LH and FSH leading to reduction in estradiol levels A 35-year-old G1P0 woman business executive with last menstrual period one week ago presents with an 8month history of pelvic pain. She reports regular menstrual cycles with moderate flow and dysmenorrhea, relieved with NSAIDs. She describes her pain as a deep, achy sensation with frequent sharp exacerbations. She has not been sexually active for the last several months because of dyspareunia and some arguments with her new partner of 1 year. She uses no steroid contraception. She has no history of sexually transmitted infections. Her medical history is significant for irritable bowel syndrome, managed with a fiber supplement. She has smoked one pack of cigarettes a day since age 25, and drinks a glass of wine three times a week. She tries to exercise regularly by running three to four times a week. This new pain is distinctly different from her IBS symptoms. Which of the following risk factors can contribute to increased incidence of pelvic pain in this patient? a) Her alcohol use b) Her smoking habit c) Her job d) Her new partner e) Her age Most published evidence suggests a significant association of physical and sexual abuse with various chronic pain disorders. The arguments with the new partner allude to possible abuse issues. Studies have found that 40-50% of women with chronic pelvic pain have a history of abuse. Whether abuse (physical or sexual) specifically causes chronic pelvic pain is not clear, nor is a mechanism established by which abuse might lead to the development of chronic pelvic pain. Women with a history of sexual abuse and high somatization scores have been found to be more likely to have non-somatic pelvic pain, suggesting the link between abuse and chronic pelvic pain may be psychologic or neurologic. However, studies also suggest that trauma or abuse may also result in biophysical changes, by literally heightening a person's physical sensitivity to pain. d) Her new partner A 48-year-old G4P4 woman with her last menstrual period 4 weeks ago presents with a 1 year history of noncyclical pelvic pain, dysmenorrhea and dyspareunia. She has a past history of endometriosis, diagnosed 10years ago by laparoscopy. She had previously been on cyclic oral contraceptives for both birth control and menstrual cycle control, but elected for permanent laparoscopic sterilization 14 months ago. Minimal endometriosis was noted at the time of laparoscopy. She now has recurrent symptoms, but wants to avoid medication. The most appropriate surgical option for this patient is: a) Hysteroscopy and dilation and curettage b) Diagnostic laparoscopy c) Hysterectomy with bilateral salpingo-oophorectomy d) Endometrial ablation e) Wedge resection of the ovaries It is estimated that chronic pelvic pain is the principal preoperative indication for 10-12% of hysterectomies. Since the patient had a tubal ligation, and does not desire any more children, the best option is removal of ovaries with or without a hysterectomy. Repeat laparoscopy with treatment of endometriosis and adhesions can be helpful; however, the patient will continue to be at increased risk of recurrent disease. An endometrial ablation or removal of ovaries alone would not be very helpful in the setting of non-cyclical pain. c) Hysterectomy with bilateral salpingo-oophorectomy A 62-year-old G5P5 grandmother, menopausal for 12 years, presents with a 7-month history of pelvic pain and pressure, as well as abdominal distention and bloating. She experiences occasional constipation, but no melena or hematochezia. She also has mild to moderate urinary frequency without dysuria, hematuria or flank pain. She describes no exacerbating or alleviating factors. Her medical history is significant for hypertension and obesity. She has never been on hormone therapy. She notes one episode of light vaginal bleeding several months ago. Her family history is significant for postmenopausal ovarian cancer in her mother and maternal aunt, but is otherwise negative for breast, endometrial or colon carcinoma. Pelvic examination is remarkable

for vaginal atrophy, cervical stenosis and difficult uterine and adnexal assessment due to her body habitus. What is the most appropriate next step in the management of this patient? a) Performing a transvaginal ultrasound b) Diagnostic laparoscopy c) Computed Axial Tomography (CT) scan of the abdomen and pelvis d) Colonoscopy e) Hysteroscopy Given the patients age, nonspecific abdomino-pelvic symptoms, recent postmenopausal bleeding episode and family history of ovarian cancer, a transvaginal ultrasound is the next best step as it is more sensitive than CT for evaluation of the uterus and adnexa. Endometrial thickness of 4 mm or less on transvaginal ultrasound in postmenopausal women is rarely associated with endometrial malignancy. Colonoscopy is useful for colorectal cancer screening, as well as evaluation of the patients gastrointestinal symptoms, but would not provide information regarding pelvic anatomy. Diagnostic laparoscopy would be a more invasive procedure that could be performed as indicated, after these other diagnostic studies. Hysteroscopy might be useful based on the ultrasound results, since it might be difficult to perform an endometrial biopsy in the office. a) Performing a transvaginal ultrasound A 29-year-old G0 woman, presents for follow up after hospitalization for pelvic inflammatory disease and right tubo-ovarian abscess (TOA) 14-months ago. She was treated with parenteral antibiotics. She now has a 10month history of pelvic pain and deep thrust dyspareunia, as well as a persistent right adnexal tubular mass on ultrasound. She desires future fertility. Pelvic examination reveals a retroverted, normal-sized uterus with limited mobility and marked tenderness and fullness in the right adnexa. Findings at the time of laparoscopy include multiple filmy and dense adhesions between the posterior uterus and cul-de-sac, normal left fallopian tube and ovary, and large right hydrosalpinx, with a few filmy adhesions between the right ovary and distended fallopian tube. What is the most appropriate treatment for this patient? a) Exploratory laparotomy, right salpingo-oophorectomy and lysis of adhesions b) Exploratory laparotomy, right salpingectomy and lysis of adhesions c) Laparoscopic right salpingo-oophorectomy and lysis of adhesions d) Laparoscopic right salpingectomy and lysis of adhesions e) Total abdominal hysterectomy and bilateral salpingo-oophorectomy Approximately 18-35% of all women with acute pelvic inflammatory disease (PID) develop chronic pelvic pain. The actual mechanisms by which chronic pelvic pain results from PID are not known and not all women with reproductive organ damage secondary to acute PID develop chronic pelvic pain. Chronic pelvic pain is the indication for at least 40% of all gynecologic laparoscopies. Endometriosis and adhesions account for more than 90% of the diagnoses in women with discernible laparoscopic abnormalities. Given this patients desire for future fertility, conservative surgical intervention is indicated with lysis of adhesions. Retention of the patients ovaries is also possible, given the limited involvement with adhesions. Since the patient has a persistent hydrosalpinx and pelvic pain, the right fallopian tube should be removed with conservation of the right ovary. A salpingostomy (an incision in the tube) would not be adequate treatment for this patient. d) Laparoscopic right salpingectomy and lysis of adhesions A 33-year-old G2P2 woman reports a 2-year history of severe dysmenorrhea, menorrhagia and pelvic pain immediately following the forceps-assisted delivery of her last child, which was complicated by postpartum hemorrhage and right vaginal sidewall laceration. She describes her pelvic pain as primarily in the right lower quadrant, radiating into the vagina. Her pain worsens throughout the day with standing and is associated with pelvic pressure and fullness. Her pelvic examination reveals a mildly enlarged uterus with marked tenderness to palpation of the right adnexa, and no other significant findings. A vaginal ultrasound with color-flow Doppler reveals multiple dilated vessels traversing the right broad ligament to the lower uterus and cervix. The uterus shows no fibroids or other significant changes. Endometrial thickness appears normal. Which of the following is the most likely diagnosis in this patient? a) Endometriosis b) Endometritis c) Adenomyosis d) Pelvic congestion e) Pelvic floor relaxation The pelvic congestion syndrome is a cause of chronic pelvic pain occurring in the setting of pelvic varicosities. The unique characteristics of the pelvic veins make them vulnerable to chronic dilatation with stasis leading to vascular congestion. These veins are thin walled and unsupported, with relatively weak attachments between the supporting connective tissue. The cause of pelvic vein congestion is unknown. Hormonal factors contribute to vasodilatation when pelvic veins are exposed to high concentration of Estradiol, which inhibits reflex vasoconstriction of vessels, induces

uterine enlargement with selective dilatation of ovarian and uterine veins. This pain may be of variable intensity and duration, is worse premenstrually and during pregnancy, and is aggravated by standing, fatigue and coitus. The pain is often described as a pelvic fullness or heaviness, which may extend to the vulvar area and legs. Associated symptoms include vaginal discharge, backache and urinary frequency. Menstrual cycle defects and dysmenorrhea are common. No signs of pelvic floor relaxation were noted on exam. d) Pelvic congestion A 45-year-old G2P2 woman underwent an abdominal hysterectomy for a large fibroid uterus via a low transverse skin incision. Her postoperative course was significant for new onset right lower quadrant pain and numbness, radiating into the right inguinal area and medial thigh. Her pain was exacerbated by adduction of her right thigh. On abdominal examination, there is a well-healed low transverse incision. Her pain is reproduced with adduction of the right thigh. There is decreased sensation to light touch and pinprick over the right inguinal area and right medial thigh. Patellar reflexes are 2+ and symmetric. Entrapment of which of the following nerves is the most likely cause of her pain? a) Obturator nerve b) Ilioinguinal nerve c) Lateral femoral cutaneous nerve d) Femoral nerve e) Iliohypogastric Nerve entrapment syndrome is a commonly misdiagnosed neuropathy that can complicate pelvic surgical procedures performed through a low transverse incision. The nerves at risk are the iliohypogastric nerve (T-12, L-1) and the ilioinguinal (T-12, L-1) nerve. These two nerves exit the spinal column at the 12th vertebral body and pass laterally through the psoas muscle before piercing the transversus abdominus muscle to the anterior abdominal wall. Once at the anterior superior iliac spine, the iliohypogastric nerve courses medially between the internal and external oblique muscles, becoming cutaneous 1cm superior to the superficial inguinal ring. The iliohypogastric nerve provides cutaneous sensation to the groin and the skin overlying the pubis. The ilioinguinal nerve follows a similar, although slightly lower, course as the iliohypogastric nerve where it provides cutaneous sensation to the groin, symphysis, labium and upper inner thigh. These nerves may become susceptible to injury when a low transverse incision is extended beyond the lateral border of the rectus abdominus muscle, into the internal oblique muscle. Symptoms are attributed to suture incorporation of the nerve during fascial closure, direct nerve trauma with subsequent neuroma formation, or neural constriction due to normal scarring and healing. b) Ilioinguinal nerve would be awarded 10 points A 68 year-old G3P3 woman comes to the office due to breast tenderness. She is in good health and not taking any medications. Family history is significant for her 70 year-old sister being diagnosed with breast cancer. On breast examination, her breasts have no lesions; there are no palpable masses, nodules or lymphadenopathy. Her last mammogram was 4 months ago and was normal. What is the most appropriate next step in the management of this patient? a) Order a mammogram b) Order a breast ultrasound c) Obtain genetic testing (BRCA-1 and BRCA-2 mutations) d) Reassurance e) Order a breast MRI Age and gender are the greatest risk factors for developing breast cancer. Having one first-degree relative with breast cancer does increase the risk, but genetic mutations occur in a low percentage of the general population. There is no indication for a mammogram since the patients last mammogram was normal 4 months ago. Ultrasound and MRI would not add valuable information especially in the setting of a normal mammogram and no masses on physical examination. Genetic testing is not indicated in this case as there is no strong family history and the sister with breast cancer was postmenopausal at time of diagnos d) Reassurance A 31 year-old woman presents at the office for an annual exam. She has no prior history of breast disease and has large, pendulous breasts. There is no family history of breast cancer. Which of the following positions enhances the visualization of asymmetry? a) Upright position of the patient b) Leaning forward c) Pressing her hands on her hips d) Left lateral decubitus position e) Right lateral decubitus position A properly performed clinical breast exam includes inspection and palpation of the breasts. Leaning forward facilitates the inspection for asymmetry even if the patient has large and pendulous breasts (this position allows the breasts to

hang from the chest wall). A clinical breast exam should include inspection of the patient in an upright, sitting position, while pressing her hands on her hips, placing hands above the head and leaning forward. Moving the pectoralis muscle elicits skin changes that can be associated with invasive breast cancer, such as skin puckering or dimpling. When the patient is supine with her hands above her head, the breast tissue rests on the chest wall and is easier to palpate for masses. If the patient has large breasts, placing a pillow under her shoulder can assist with maintaining an even distribution of the breast tissue on the chest wall. Differences between the two breasts, such as differences in breast contour or sizes, nipple inversion or retraction, may be other physical findings suggestive of an underlying mass. Inspection of the patient in the lateral decubitus position is not usually helpful. b) Leaning forward A 48 year-old woman presents with complaints of a white, watery nipple discharge for 4 months. She has been told in the past she had fibrocystic breast changes, but otherwise has no significant medical problems or surgical history. The white nipple discharge is noted on manual expression, but the exam is otherwise normal. Her serum prolactin level was 45 ng/ml (normal below 40 ng/ml). What is the most appropriate next step in the management of this patient? a) Obtain a brain MRI b) Obtain a - HCG c) Begin Bromocriptine d) Obtain a fasting prolactin level e) Order a ductogram Stimulation of the breast during the physical examination may give rise to an elevated prolactin level. Accurate prolactin levels are best obtained with patients fasting. If these are still elevated, then a brain MRI would be indicated to rule out a pituitary tumor. Although pathologic factors, such as hypothyroidism, hypothalamic disorders, pituitary disorders (adenomas, empty sella syndrome), chest lesions (breast implants, thoracotomy scars, and herpes zoster) and renal failure can elevate prolactin levels, a non-significant benign elevation needs to be ruled out first. A ductogram is usually indicated in patients who have bloody discharge from the nipple. d) Obtain a fasting prolactin level A 42 year-old G3P3 patient comes to the office after noticing a breast mass while performing a breast selfexam. She is in good health and has normal menstrual cycles. Family history is significant for multiple first and second degree relatives having breast cancer. Physical exam reveals a 2-centimeter dominant breast mass. The remainder of the exam is normal. A mammogram obtained today shows no abnormalities. What is the most appropriate next step in the management of this patient? a) Reassurance and observation b) Obtain genetic testing c) MRI of the breast d) Fine needle aspiration e) Repeat mammogram in 2 months Any solid dominant breast mass on exam should be evaluated cytologically, with a fine needle aspiration (FNA) or histologically, with an excisional biopsy. In this scenario, an MRI should not be part of the initial work-up for the patients palpable breast mass. Testing for genetic mutations is indicated for patients with a strong family history of breast cancer, but diagnosis is the most important next step in the management of this patient. A normal mammogram does not rule out the presence of cancer, and there is no reason to repeat the mammogram in two months, especially considering that the first one was normal. d) Fine needle aspiration A 24 year-old patient complains of cyclic mastalgia since the onset of her period at age 12. The symptoms have increased over the years and were less troublesome when she took oral contraceptives a few years ago. Currently, she takes no medications and is not sexually active. She is a strict vegetarian and eats soy products. She does not smoke and reports she drinks a glass of wine three times a week, and several diet cokes every day. Her mother was diagnosed with breast cancer at age 55. Her breast exam is normal, except for some fibrocystic changes. Which of the following elements in her history contributes to her increasing pain? a) Alcohol intake b) Vegetarian diet c) Family history of breast cancer d) Caffeine intake e) Age at menarche Fibrocystic breast changes are the most common type of benign breast conditions and occur most often during the reproductive years. There is an increased risk of breast cancer when atypia is present. The changes do not appear

distinct histologically (3 stages) or mammographically. Fibrocystic disease is often associated with cyclic mastalgia, possibly related to a pronounced hormonal response. Caffeine intake can increase the pain associated with fibrocystic breast changes, so recommending that she decrease her caffeine intake might be helpful. d) Caffeine intake A 54 year-old woman presents with a breast mass she noticed two months ago. She has no family history of breast cancer. On exam, there is a 2 cm mass palpable in the upper outer quadrant of the left breast. There are no other masses noted and no lymphadenopathy. A fine needle aspiration returns bloody fluid and reduces the size of the mass to 1 cm. In addition to obtaining a mammogram, what is the most appropriate next step in the management of this patient? a) Repeat exam in 2 months b) Excisional biopsy of the mass c) Obtain a breast MRI d) Perform a lumpectomy and lymph node dissection e) Follow-up in 1 year if mammogram is normal The first noticeable symptom of breast cancer is typically a lump that feels different from the rest of the breast tissue. More breast cancer cases are discovered when the woman feels a lump. . Breast cancer can also present with a spontaneous bloody nipple discharge. Even though the mass decreased in size after aspiration, the bloody discharge obtained obligates an excisional biopsy to be performed to rule out breast cancer. If clear discharge is obtained on aspiration and the mass resolves, reexamination in 2 months is appropriate to check that the cyst has not recurred. An MRI is not the appropriate next step and lumpectomy with lymph node dissection is not yet indicated in this case. A normal mammogram does not rule out breast cancer, especially in the presence of bloody discharge. b) Excisional biopsy of the mass A 23 year-old nulliparous woman presents with a painful mass in her axilla for three days. She has no personal or family history of breast disease. On exam, no abnormalities are seen on inspection and no breast mass is palpated. A tender, enlarged lymph node is noted in her left axilla. What is the most likely underlying etiology for the lymph node enlargement? a) Early stage carcinoma b) Fibroadenoma c) Fibrocystic change d) Infection e) Duct obstruction Although axillary lymphadenopathy can be a presenting sign for breast cancer, a tender mobile axillary lymph node is more typically associated with an infectious process. Adenopathy associated with carcinoma is usually firm, non-tender and fixed. Fibroadenomas are common, but are usually firm, painless and freely movable. A clogged milk duct can be present in the axillary region, but it is typically present in a woman who is breast feeding. d) Infection A 25 year-old G1P1 comes to the office due to left breast pain and fever. She is breast feeding her 2 1/2-week old infant. The symptoms began earlier in the day and are not relieved by acetaminophen. Blood pressure 120/60; pulse 64; temperature 99.9 F, 37.7 C. On exam, she has erythema on the upper outer quadrant of the left breast, which is tender to touch; there are no palpable masses. In addition to starting oral antibiotics, what is the most appropriate next step in the management of this patient? a) Discontinue breastfeeding b) Add ibuprofen for pain relief c) Obtain a breast ultrasound d) Use a topical antifungal e) Admission to the hospital Mastitis that accompanies pregnancy or nursing is the most common breast infection. Puerperal mastitis most commonly occurs during the second to fourth week after delivery. Patients are treated with oral or IV antibiotics, depending on the severity of infection. Patients may use ibuprofen in addition to acetaminophen for pain relief, and are encouraged to continue breast feeding or expressing their milk during treatment. Mastitis is usually treated as an outpatient. Patients may be admitted to the hospital in severe cases and will be administered intravenous antibiotics. A breast ultrasound is not indicated if there is no suspicion of a breast abscess. b) Add ibuprofen for pain relief A 25 year-old G1P1 who is breast feeding her 2 week old comes to the office with left breast pain and fever. The symptoms began earlier today and are not relieved by acetaminophen. She has no known drug allergies. Blood pressure 120/60; pulse 64; temperature 99.9 F, 37.7 C. On exam, there is erythema on the upper outer

quadrant of the left breast, which is tender to touch; there are no palpable masses. What is the most appropriate antibiotic therapy for this patient? a) Dicloxacillin b) Erythromycin c) Doxycycline d) Gentamicin e) Cefotetan Most postpartum mastitis is caused by staphylococcus aureus, so a penicillin-type drug is the first line of treatment. Dicloxacillin is used due to the large prevalence of penicillin resistant staphylococci. Erythromycin may be used in penicillin-resistant patients. a) Dicloxacillin A 42 year-old G3P3 patient comes to the office after noticing a breast mass while performing a breast selfexam. She is in good health and has normal menstrual cycles. Physical exam is significant for a 2 cm dominant breast mass. The remainder of the exam is normal. A mammogram obtained today shows no abnormalities. A fine needle aspiration was negative, and the mass persisted. What is the most appropriate next step in the management of this patient? a) Reassurance and observation b) Obtain CAT scan of the chest c) Breast ultrasound d) Perform an excisional biopsy e) Repeat mammogram in 2 months A specimen obtained on fine-needle aspiration is examined both histologically and cytologically. An excisional biopsy should be performed when the results are negative, due to the possibility of a false-negative result. It can, however, prevent the need for other diagnostic testing and is the appropriate next step. Breast ultrasound can be used to distinguish between a cyst and a solid mass. Fine needle aspiration under ultrasound guidance can help distinguish a fibroadenoma from a cyst and exclude cancer in certain situations. A normal mammogram does not rule out breast cancer and there is no need to repeat it in 2 months. There are no indications for obtaining a CAT scan of the chest in the initial diagnosis of this patient. d) Perform an excisional biopsy An 18 year-old college student is seen in the emergency room, claiming she was raped by a 29 year-old janitor in her dorm 4 hours ago. He threatened her with a knife and she did not resist. She appears calm and has a flattened affect when the history is taken. Currently, she is sexually active with a fellow student and is taking birth control pills. The alleged attacker used a condom, which she helped him put on. The student is 52 tall and weighs 110 pounds. She is a cheerleader for the college football team. Examination reveals no bruising and gynecologic examination reveals no apparent injuries. Which of the following statements is the most likely explanation for this patients presentation? a) The attack was not emotionally traumatic b) The student is in shock c) The attack never happened d) The student cooperated with the attacker e) The student suffers from chronic depression Women who are raped are often in denial or shock. A woman may shower after an attack, destroying evidence in an attempt to deny what happened or to clean herself. She may blame herself for the attack and feel she should have resisted more. b) The student is in shock A 10 year-old girl has been drawing sexually explicit pictures during art class. Her behavior in class has ranged from being shy to being aggressive with her fellow students. She wont go to the bathroom unless someone goes with her. Physical examination shows a normally developed girl. Breast budding is occurring and she has wisps of axillary and pubic hair. She will not let you look at her bottom. Which of the following is the most likely cause of this condition? a) Normal adolescent development b) Attention deficit disorder c) Precocious puberty d) Child abuse e) Hypothyroidism Children who are sexually abused will often act out and behave inappropriately. There is often no evidence of physical injury and a careful history must be taken. d) Child abuse

A 6 year-old girl has begun to bed-wet after having been toilet trained for 3 years. She has also been found playing with herself for the past several weeks. In a private conversation the child states, My daddy has been playing with me down there. Which of the following is the most appropriate course of action at this time? a) Suggest family counseling b) Notify police c) Treat for urinary tract infection d) Speak with both parents and child together e) Refer to a pediatric urologist Child abuse can happen in any circumstance. It is not always the boyfriend who commits such a crime. Often the perpetrator is a pillar of the community. The child must be able to talk in a safe environment, certainly not in the presence of the person accused of the deed. b) Notify police A mother brings in her 6 year-old daughter for an examination. She states that she thinks her boyfriend may be fooling around with the child. The girl runs and hides when the boyfriend comes to their home. Examination reveals a normal 6 year-old girl. There is no evidence of sexual abuse. The mother wants nothing done as she is dependent on her boyfriend for support. Which of the following is the most appropriate course of action? a) Advise return visit in one month b) Notify police c) Advise family counseling d) Inform mother that the child may be jealous e) Perform colposcopic examination In all suspected cases of child abuse, the proper authorities must be notified. Often victims of child abuse have no physical findings. Colposcopic examination is often inconclusive. The child may be acting out, but it is better to err on the side of safety. b) Notify police A 3 year-old girl is taken to see her primary care physician by her father, secondary to vaginal discharge and swelling of the vulvar area for the past week. She has been scratching the area and says it hurts. She was on an antibiotic 2 weeks ago for an ear infection. The child has had normal growth and development. She has 2 male siblings ages 8 and 6. Her father is a stay-at-home dad and her mother is an attorney. Examination reveals a red and swollen vulva with linear abrasions over the area. Which of the following is the most appropriate next step in treatment? a) Check a complete blood count b) Begin antifungal treatment c) Perform nasal speculum vaginal examination d) Refuse to treat until mother is present e) Notify police Yeast infections are common after antibiotic therapy. Scratching can appear like abuse is taking place. To examine a child, even with a nasal speculum is traumatic. If a foreign body is suspected, an exam under anesthesia may be necessary, if ultrasound is not successful or indicated. b) Begin antifungal treatment A 4 year-old girl is taken to see her pediatrician by her father. He states that he has noticed a foul odor in the area of the childs private parts. He brings in a pair of panties with a brown-yellow stain that has a funny smell. The girl is otherwise healthy. Which is the most likely cause of this condition? a) Child abuse b) Foreign body c) Yeast infection d) Pinworm infection e) Urinary tract infection Children will often place foreign objects in any body orifice. Toilet paper is the most common foreign body found. b) Foreign body A 3 year-old girl is seen in the emergency room because her mother saw her putting something inside of her vagina. The mother had placed an earring stud on the bed and the child was playing with it. Ultrasound

examination reveals a metal object deep in the childs vagina. Which of the following is the most appropriate course of action at this time? a) Vaginal extraction using a nasal speculum for visualization b) Removal under anesthesia c) Provide instructions to the girl and mother on how to remove the object d) Prescribe broad speculum antibiotic until object expelled e) Order gentle vaginal douching to assist in expelling object An attempt to remove an object without anesthesia is almost impossible in such a young child. Under anesthesia, a gentle rectal exam using the smallest finger can often milk-out a foreign body. b) Removal under anesthesia An 18 year-old college student is brought to your office by her roommate who found her sobbing in their room. The student states that she was sitting on a couch with her new boyfriend. He gave her something to drink. She thinks that she passed out and awoke 2 hours later and was undressed. She is afraid that she may have had intercourse. She is not on any form of contraceptive. Her last menstrual period was 2 weeks ago. Examination reveals no obvious injuries and a vaginal swab shows motile sperm. In addition to providing counseling and screening for sexually transmitted infections, which of the following is the most appropriate next step in management? a) Discuss her responsibility to report this to her college b) Admit to the hospital for intravenous antibiotics c) Recommend emergency contraception d) Notify her college advisor e) Obtain more information from her roomate In a case of date rape, the best course of action is to ensure that the patient does not get pregnant. The patient should also have screening for sexually transmitted diseases, with consideration to being offered antibiotic prophylaxis. Although the risk of infection is unknown among victims of sexual assault, it may be higher compared to consensual sexual encounters. c) Recommend emergency contraception A 17 year-old G0 was kissing her 18 year-old boyfriend in a parked car. She refused to have sexual intercourse and he overpowered her, hit her on the face and raped her. He did not use a condom and ejaculated inside her. Which of the following made this case a rape? a) Failure to use effective barrier contraception b) Sexual intercourse c) Lack of consent d) Legal age of woman e) Legal age of the boyfriend Rape is characterized by lack of consent or inability to give consent. Most definitions include the use of physical force, deception, intimidation or the threat of bodily harm. c) Lack of consent A 30 year-old G2P2 was raped by a stranger while she was walking through a park alone at 2:00 am. The assailant did not use a condom. An intern evaluated the patient in the emergency room. He took a detailed history, performed a complete physical exam and collected forensic specimens. He obtained cultures for gonorrhea and chlamydia, and obtained an RPR, hepatitis antigens, an HIV test, a urinalysis and culture, and a pregnancy test. He provided the patient with postcoital contraceptive medication. Which of the following additional actions is most appropriate? a) Notify police b) The patients parents or her closest relative or friend should be notified c) The patient should be offered antibiotic prophylaxis for sexually transmitted infections d) The patient should be counseled regarding the practice of safe behavior, including not walking in deserted areas alone e) The patient should have testing for herpes simplex virus (HSV) Antibiotic prophylaxis should be offered to all adult rape victims. Although patients are often reticent to do so, they should be gently encouraged to work with the police. This has been associated with improved emotional outcomes for victims. Counseling the patient regarding the practice of safe behavior at this time may make the patient feel blame, when the blame should be placed on the rapist. HSV antibody testing is not indicated. c) The patient should be offered antibiotic prophylaxis for sexually transmitted infections A representative from a domestic violence outreach program asks for your help in distributing information, to include a hot line to call, if needed. Where is the best place for this information to be made available?

a) Office waiting room b) Distribution to all patients at time of check in c) Office rest room d) Letters sent to all your patients e) Posters in front of your office The best place to have literature is where there is the most privacy. In the other areas cited, an abuser may see the information and prevent his spouse or partner from obtaining it. c) Office rest room An 84 year-old widow is seen in the emergency room because of a pain in her right arm. Her daughter, with whom she lives, states that her mother is always falling down and is getting forgetful. She would put her in a nursing home, but her mothers social security check is needed for family expenses. The patient is on Zolpidem 10 mg for sleep. She has been unable to afford the bisphosphonates needed for bone loss. Examination reveals a poorly nourished white woman, 52 tall, weighing 94 pounds. She has tenderness in her right forearm with swelling. She is also noted to have old and new bruises on her chest and arms. An x-ray of the right arm reveals a spiral fracture. Which of the following is the most likely cause of the bone fracture? a) Osteoporosis b) Domestic violence c) Bone metastasis from occult breast cancer d) Malnutrition e) Hypocalcemia Spiral fractures rarely occur unless violence has been practiced. It is due to a twisting of the arm causing the almost pathognomonic finding. Care for the elderly is difficult, but there is no excuse for violence against the elderly. b) Domestic violence A 79 year-old woman is seen in the emergency room in a comatose state. Her daughter states that her mother is always wandering away from the home. She has tried to tie her to her bed, but has found that if she gives her Zolpidem every 4 hours, it keeps her sedated. The daughter works during the day and would put her mother in a nursing home, but she needs the mothers social security check to maintain the household. Examination reveals an unresponsive, thin, dehydrated woman with acute and chronic bedsores. After hydration and treatment of her skin condition in the hospital, which of the following is the most appropriate next step? a) Notify social services b) Notify social security c) Discharge patient home after giving daughter appropriate instructions on how to care for her elderly mother d) Advise the daughter to change her work schedule e) Decrease the Zolpidem dose In a case of obvious elderly abuse, social services must be notified. The patient should not be taking Zolpidem, and this should also be discontinued. Physicians must be aware of the social services available for their patients. Although the patient may ultimately be discharged home, this needs to be done in consultation with social services. a) Notify social services A 24 year-old woman comes to her physician for help with her premenstrual syndrome symptoms. She complains of not being herself for 3 to 4 days before her period and has episodes of crying and irritability. She denies depressive symptoms and notes she is a stay-at-home mother for her three children. After a complete history and physical examination, the patient is prescribed a selective serotonin reuptake inhibitor but, after three months, she returns as there is no change in her symptoms. Upon further discussion, the patient admits that her husband has a bad temper at times. Physical examination is normal with the exception of some bruising on the patients arms. She claims that she fell and that she is often clumsy. Which of the following is the most appropriate next step in the management of this patient? a) Report the injuries to the police b) Offer domestic violence resources to the patient c) Refer the patient for a psychiatric consultation d) Arrange for a home visit by a womens shelter counselor e) Arrange for security to escort her to a womens shelter Victims of domestic violence frequently present with vague physical and emotional complaints that are consistent with PMS. Victims will rarely volunteer information on first physician encounters, but subsequent visits give opportunities to offer resources for assistance. It is the patients choice as to whether she would like to involve the police. b) Offer domestic violence

A 28 year-old G0 underwent a routine Pap smear 6 weeks ago which showed low grade squamous intraepithelial lesion (LGSIL) with HPV associated changes. She currently has normal menses, no pelvic pain and no general complaints. Her previous medical history is negative. She has no previous abnormal Pap smears and no history of sexually transmitted infections. She has smoked -pack of cigarettes per day for 10 years; no alcohol or drug use. She has had 3 sexual partners in the past. On examination, she has a normal appearing cervix and normal bimanual exam. What is the most appropriate next step in this patients care? a) Treat with antibiotics and repeat the Pap Smear b) Loop Electrosurgical Excision Procedure (LEEP) c) Cold knife cone biopsy d) Colposcopy e) Repeat Pap smear today The most recent consensus guidelines (2006) state that management of LGSIL (unless the woman is pregnant, postmenopausal or an adolescent) is initial colposcopic examination. An excisional procedure, such as cold knife biopsy or LEEP, is not warranted without a tissue diagnosis of dysplasia. The Pap smear is merely a screening tool and, as such, cannot formulate a definitive diagnosis. In fact, up to 20% of patients with LGSIL on Pap smear have HGSIL on colposcopically-directed biopsy. In contrast, up to 50% of patients with LGSIL on Pap smear have a negative colposcopy. Antibiotic treatment is not warranted unless there is an infection. If a Pap smear is repeated prior to 6-8 weeks following the last one, reparative changes may still be happening to the cervix. This reduces the ability of the test to be a good screening tool. (The false positive and/or false negative rates can be affected). In some instances, LGSIL may be followed with serial Pap smears; however, a tissue diagnosis or a colposcopy without evidence of HGSIL changes must be done initially. d) Colposcopy A 34 year-old G2P2 woman who recently underwent colposcopy with biopsy following a Pap smear which showed low-grade squamous intraepithelial lesion, comes in to discuss her results. The biopsy results showed HGSIL (CIN III). Endocervical curettage showed benign endocervical cells. What is the most appropriate next step in the management of this patient? a) Radical hysterectomy b) Simple hysterectomy c) Repeat Pap smear in 4 months d) Human Papilloma Virus (HPV) testing e) Loop Electrosurgical Excision Procedure (LEEP) The LEEP procedure is usually done in the office under local intracervical anesthesia. It involves using an electrosurgical unit (similar to the Bovie in the operating room), along with a wire loop of varying sizes to remove the entire transformation zone and the dysplastic area(s) identified during colposcopy. This tissue is then sent to pathology so that the area of dysplasia can be fully evaluated. A radical hysterectomy is an option for patients with invasive cervical carcinoma stage Ia2 through II a. This patient has only cervical dysplasia (not invasive cancer). A simple hysterectomy is not needed, as it is more involved than a LEEP. It is not generally recommended to conservatively offer serial Pap testing for patients with biopsy proven CIN III or HGSIL. Although spontaneous regression of dysplasia may occur with a high-grade lesion, the rate of regression is much lower than with LGSIL. On the other hand, the progression rate of HGSIL to invasive cancer is much higher (up to 12%). High risk HPV testing can be recommended as an adjunct to Pap smears such as with ASCUS Pap smears. It would not offer further information in this patients scenario. e) Loop Electrosurgical Excision Procedure (LEEP) A 50 year-old G3P3 female comes to the office due to heavy periods, spotting between menses, fatigue and weakness. Over the past 6-8 months, she has noticed a significant increase in the amount of her menstrual bleeding, currently requiring a box of 30 pads for each month. She has noticed an increase in the amount of blood clots and cramping pain during menses. Her previous history is significant for hypertension for 10 years, controlled with hydrochlorothiazide and a postpartum bilateral tubal ligation 20 years ago. She has no history of abnormal Pap smears and no sexually transmitted infections. Blood pressure is 138/84; pulse 82; respirations 20; weight 220 pounds; height 68. On pelvic exam, uterus is approximately 10-week size. She is nontender on bimanual exam and no adnexal masses are appreciated. A Pap smear 1 month ago was normal, and her hematocrit is 30. What is the next best step in the management of this patient? a) Hysterectomy b) Endometrial biopsy c) Erythropoietin d) Blood transfusion e) Endometrial ablation Correct!!! Endometrial biopsy is typically an office procedure which does not cause extreme discomfort for the

patient. It results in information necessary to tailor the patients care, such as presence of endometritis, endometrial polyps or endometrial carcinoma. In a patient with significant risk factors for endometrial carcinoma, this should be done prior to a hysterectomy or ablation, if at all possible. A hysterectomy or endometrial ablation would be incorrect, as further workup is needed prior to taking the patient for one of these procedures. In addition, this patient, potentially, has a treatable condition, such as endometritis, an endometrial polyp, endometrial hyperplasia or an enlarging submucosal fibroid, which could all be treated with either medical therapy or a less radical procedure. With stable vital signs and a hematocrit of 30, erythropoietin and a blood transfusion would not be indicated for this patient. Her anemia could be corrected using ferrous sulfate over a period of a few months, along with targeted therapy to decrease her vaginal bleeding (therapy would be based on the endometrial biopsy result). b) Endometrial biopsy A 50 year-old G4P4 woman presents for her yearly checkup. She states her menses are of normal flow every 32 days, with minimal cramping and 5-day duration. Review of systems is negative. She has no medical problems. She had a bilateral tubal ligation following her last child, and a laparoscopic cholecystectomy 5 years ago. She has a history of LGSIL Pap smear with colposcopy and cryotherapy 15 years ago; her Paps have been normal since then. She does not smoke, drink alcohol or use any drugs. She is sexually active with one partner with no problems. Her general exam, including a breast exam and pelvic exam, is normal. In addition to performing a Pap smear, which of the following is the most appropriate screening test for this patient? a) Pelvic ultrasound b) Endometrial biopsy c) Mammogram d) DEXA scan e) Colposcopy Correct!!! Women should be offered mammograms yearly starting at age 50 and at least every 2 years between age 40 and 50. Ultrasound is not a good screening modality for pelvic pathology. It may be used along with yearly examination if the patients habitus does not allow full evaluation of the adnexa; this patient had a normal exam, so it is not indicated. An endometrial biopsy is indicated if a patient is experiencing irregular bleeding. A DEXA scan is not indicated in a pre-menopausal patient. A colposcopy is not indicated as she has not had a recent history of abnormal Pap smears. c) Mammogram A 26 year-old G0 was found to have a low-grade squamous intraepithelial lesion (LGSIL) on routine Pap smear. She underwent a colposcopy with cervical biopsy. Her colposcopy was adequate and biopsy results showed CIN-I. There was no endocervical glandular involvement. Endocervical curettage showed benign cells. Which of the following is the most appropriate treatment for this patient? a) Cold knife conization b) Loop Electrosurgical Excision Procedure (LEEP) c) Cryotherapy d) Follow up Pap smear in 6 months e) Complete removal of the lesion Correct!!! The patient should be followed with Pap smears at 6 and 12 months or undergo HPV DNA testing at 12 months. Excisional or ablative procedures are not indicated for LGSIL. Indications for cold knife conization (CKC) include: positive endocervical curettage, HGSIL lesion either too large for LEEP, patient not tolerant of examination in office, lesion extending into the endocervical canal beyond vision, or to rule out invasive cancer (classify the depth of invasion if biopsy shows invasion). It is unusual to manage low grade lesions by CKC. Indications for LEEP are similar to CKC. d) Follow up Pap smear in 6 months A 25 year-old woman presents complaining of a wart on her vulva for one year. The area has been treated several times in the past year with several applications of trichloroacetic acid in the office, as well as a 12week course of Imiquimod cream at home. The lesion never fully resolved with either treatment. On examination, an area of verrucous lesions approximately 1 cm x 1.5 cm is noted on the left labia majora near the fourchette. Upon application of 5% acetic acid, the area becomes acetowhite. There are no other colposcopic abnormalities. Remainder of the exam is normal. A Pap smear 6 months ago is normal. What is the most appropriate next step in the management of this patient? a) Continued treatment with Imiquimod b) Repeat Pap smear today c) Laser vaporization of the lesion

d) Vulvar biopsy of lesion e) Interferon injection Correct!!! A vulvar lesion unresponsive to treatment needs a biopsy. In addition to testing for invasive cancer, the biopsy can also ensure that your diagnosis and treatment are correct. If the initial diagnosis of condyloma is unsure, a biopsy should be performed prior to initiating therapy. Imiquimod would not be recommended, as this patient previously had a full treatment without total response. Prior to initiating treatment again, a tissue diagnosis is recommended. A repeat Pap is not indicated for a vulvar lesion, and, prior to using laser vaporization to destroy the lesion, a biopsy should be done to ensure that the lesion is not cancer. Interferon is not effective in the treatment of HPV. d) Vulvar biopsy of lesion A 30 year-old woman comes to the office because she desires removal of her IUD. She had a Paragard (Copper T) Intrauterine Device placed for contraception 4 years ago and now she desires pregnancy. She is in good health and has no history of abnormal Pap smears or sexually transmitted infections. Pelvic exam shows a normal appearing cervix and no IUD string visible. Ultrasound shows the IUD in the uterine cavity. An attempt is made to remove the IUD with an IUD hook and failed. What is the most appropriate next step in the management of this patient? a) In vitro fertilization b) Laparoscopy c) Endometrial biopsy d) A pelvic MRI e) Hysteroscopy Correct!!! A hysteroscopy would be easily performed either in the office or in the operating room, and the IUD could then be removed under direct visualization. This would be the best choice for this patient. In vitro fertilization requires a normal endometrial cavity so that the embryo may implant in the uterus. A retained IUD is not an indication for this procedure. Having an ultrasound showing the retained IUD in the uterine cavity, it would not be visible upon laparoscopy. If the IUD had been seen outside the uterus, laparoscopy could be offered for removal of the IUD. A pelvic MRI will not give any additional information what would be helpful in the management of this patient. e) Hysteroscopy A 42 year-old woman presents to the office with a breast mass. She noticed a firm, slightly tender mass on her right breast during a self breast exam 2 months ago. She has not noticed nipple discharge or skin changes. It has not enlarged nor decreased in size. The review of systems is negative. She does not use tobacco, alcohol or drugs, and she drinks 5 cans of caffeinated soda per day. She has a maternal aunt with breast cancer, who was diagnosed at age 50. On examination, the left breast is normal; right breast has a 1 cm x 2 cm rubbery mobile mass in the upper outer quadrant. There was no nipple discharge, no visible skin changes, and no axillary or supraclavicular lymphadenopathy. A mammogram three months ago was normal and showed dense breast tissue bilaterally. What is the best next step in the management of this patient to help determine her diagnosis? a) Fine needle aspiration of the mass b) Observation x 6 months and reexamine c) Repeat mammogram d) Excisional biopsy of the mass e) Decrease caffeine intake and reexamine Needle aspiration of a palpable breast mass or lymph node allows for pathologic diagnosis of the mass with minimal discomfort to the patient. Results correlate well with excisional biopsy results. Observation or waiting for the patient to decrease caffeine intake would not be recommended for a patient with a new finding of a palpable breast mass, especially in a patient with a family history of breast cancer. A mammogram does not need to be repeated, since one was performed 3 months ago. Excisional biopsy is not necessary at this point although she might ultimately require it.

A 42 year-old woman presents with chronic pelvic pain. For the past 3 years, the patient has had worsening cyclic pelvic pain along with dyspareunia. Her pain has been unresponsive to oral contraceptive pills (given in both continuous and cyclic fashion), Depo-Provera injections, and slightly responsive to Lupron (GnRH agonist) injections. She had a laparoscopy one year ago, which showed endometriosis. She is currently taking NSAIDS and narcotic analgesics for pain relief. She has never been pregnant in the past and does not desire fertility. She has no history of abnormal Pap smears or sexually transmitted infections. Pelvic exam shows normal external genitalia, vagina and cervix without lesions; no discharge noted. There is some uterine and bilateral adnexal tenderness on bimanual exam, with uterosacral nodularity noted bilaterally. Uterus is normal

size and retroverted. A pelvic ultrasound is normal. The patient desires definitive treatment. What is the next best step in the management of this patient? a) Laparoscopy with excision of endometriosis b) A radical hysterectomy c) Levonorgestrel IUD d) Hysterectomy with bilateral salpingo-oophorectomy e) Endometrial ablation A hysterectomy with bilateral salpingo-oophorectomy is the definitive treatment for a patient with pelvic pain due to endometriosis. In 60% of cases, when a patient with endometriosis undergoes a simple hysterectomy without bilateral salpingo-oophorectomy for pelvic pain, re-operation for continued pain will be necessary. Even if the patient requires hormone replacement therapy postoperatively, her pain is unlikely to return. A laparoscopy is indicated in the workup of pelvic pain in order to determine the etiology of the pain. If endometriosis is noted, it may be excised, fulgurated or burned by laser. This may offer some relief of the patients pain; however, relief is usually temporary in a premenopausal female. In addition, this patient had a previous laparoscopy with only temporary relief. A radical hysterectomy, usually used to treat cervical cancer, is too invasive for the treatment of endometriosis. It is very unlikely in this scenario to decrease the patients pain simply by changing her oral contraceptive pill. An endometrial ablation is an acceptable treatment for menorrhagia and will likely not be helpful in this patient. Although Levonorgestrel IUD may effectively relieve endometriosis, it is unlikely to do so in this patient who has been on progestins and oral contraceptives without relief. d) Hysterectomy with bilateral salpingo-oophorectomy A 38 year-old G1P1 woman comes to the office for an annual exam. She has noticed some urinary frequency over the past month. She has no dysuria, hematuria, urgency or incontinence. She has normal cycles, no history of abnormal Pap smears or sexually transmitted infections and is sexually active, with 1 partner. She smokes a quarter of a pack of cigarettes daily, and drinks one glass of wine per day. Her mother had breast cancer at age 30. Her general examination is normal. On pelvic exam, she has normal external genitalia; vagina and cervix are without lesions. Her uterus is normal size, anteverted and nontender. Her left adnexa is normal, right adnexa has a mobile, slightly tender 4 cm mass. Laboratory results show a normal urinalysis, a negative urine pregnancy test and a normal Pap smear. What is the most appropriate next step in the management of this patient? a) Perform a transvaginal ultrasound b) Perform a diagnostic laparoscopy c) Recommend a CT-guided drainage of the mass d) Order a KUB plain film e) Perform an exploratory laparotomy A transvaginal ultrasound would be the best way to begin a workup for an incidental finding of an adnexal mass. Using this modality, one can distinguish an adnexal mass from other structures, as well as note the characteristics of the mass (simple vs. complex, solid vs. cystic, thin or thick walled, size, structures involved). Using this information, a management plan may be constructed. In a stable, asymptomatic patient, laparoscopy or laparotomy would not be indicated for diagnosis until further imaging studies had been done. A CT-guided drainage would not be a good choice until one has a better understanding of the characteristics of the mass. If the mass was malignant, draining it in this manner could not only seed the needle tract, but it could also spill intra-abdominally, causing the malignancy to spread. A plain film would not be helpful unless the mass contains calcium and thus would not give additional data. a) Perform a transvaginal ultrasound would be awarded 10 points A 10 year-old girl goes to the doctors office for her regular check-up. She is healthy, active in school sports and gets good grades. On examination she is 5 feet and weighs 80 pounds. She has Tanner stage 2 breasts and no pubic hair. She asks you when she might have her first menstrual period. You explain to her that appropriate sexual development occurs in which of the following orders? a) Thelarche-Menarche-Adrenarche-Growth spurt b) Thelarche-Adrenarche-Growth spurt-Menarche c) Thelarche-Growth spurt-Menarche-Adrenarche d) Thelarche-Growth spurt-Adrenarche-Menarche The normal and predictable sequence of sexual maturation proceeds with breast budding, then adrenarche (hair growth), a growth spurt and then menarche. Breast development begins around the age of 10 and average age of menarche is 11.75 years earlier for heavier girls and later for thinner, physically active girls. b) Thelarche-Adrenarche-Growth spurt-Menarche would be awarded 10 points A 16 year-old girl goes to the doctor to discuss why she has not had a menstrual cycle. She is healthy, active on multiple sports teams. She studies hard and gets good grades in school. She has a good relationship with her parents. On examination she is 51 tall and weighs 84 pounds. Breast and pubic hair growth are at a

Tanner Stage II. External genital examination is normal. What is the most likely reason this patient has not had any menses? a) Inadequate body weight b) Poor nutrition c) Inadequate sleep d) Vitamin D deficiency e) Familial reasons There are three known critical elements for secondary sexual characteristics adequate body weight, sleep and optic exposure to sunlight. These factors especially can delay the onset of menarche. A body weight of 85 to 106 pounds is needed before menses begins. a) Inadequate body weight would be awarded 10 points A mother brings her 16 year-old daughter to the doctor because she has not begun menses and has not kept up in growth with her friends. She performs well in school and participates in extracurricular activities. On physical examination, she is 51 tall, 100 pounds and has Tanner stage 1 breast and pubic hair growth. Her breast nipples are widely spaced with a shield chest and neck is thickened. No genital tract abnormalities are noted on exam. Which of the following is the most likely cause of her delayed sexual maturation? a) Partial deletion of the long arm of the X chromosome b) Mullerian agenesis c) Down Syndrome d) Turner syndrome e) Rokitansky-Kuster-Hauser Syndrome The genetic defect of Turners syndrome is the absence of one of the X chromosomes. These females will have failure to establish secondary sexual characteristics, and have short stature and characteristic physical features pterygium colli, shield chest and cubitus valgus. Partial deletions of the long arm of the X chromosome also cause premature ovarian failure. The average age of puberty in females with Down Syndrome is not significantly different than normal females. Rokitansky-Kuster-Hauser Syndrome causes vaginal and uterine agenesis and is not suspected in this case due to the physical exam findings. d) Turner syndrome would be awarded 10 points A 16 year-old girl comes to the doctor to discuss contraception. She recently became sexually active and states she has never had a menstrual cycle. She regularly attends school and participates in the band. On physical examination, she is 53 and weighs 130 pounds. She has no secondary sexual characteristics with normal appearing external genitalia. The physician suspects Kallmann syndrome. Which of the following diagnostic tests will help confirm the diagnosis? a) An MRI of the pituitary b) Olfactory challenge c) Measurement of testosterone levels d) Pelvic ultrasound e) Cortisol levels Kallmann syndrome is characterized by olfactory tract hypoplasia and the arcuate nucleus does not secrete GnRH. Therefore these females have no sense of smell and do not develop secondary sexual characteristics. The diagnosis is often one of exclusion found during the workup of delayed puberty. The presence of anosmia with delayed puberty should suggest Kallmann syndrome. Treatment is pulsatile GnRH therapy. b) Olfactory challenge would be awarded 10 points A 7 year-old is undergoing evaluation for vaginal bleeding. On physical examination, she has Tanner III stage breasts, tall stature and an otherwise normal examination. An MRI of the brain and a pelvic ultrasound are normal. LH and FSH levels are in the pre-pubertal levels and she has normal DHEAS and androgen levels. What is the most likely diagnosis in this patient? a) Pituitary adenoma b) Congenital adrenal hyperplasia c) True precocious puberty d) Testicular feminization e) Ovarian neoplasm True precocious puberty is a diagnosis of exclusion where the sex steroids are increased by the hypothalamicpituitary-gonadal axis, with increased pulsatile GnRH secretion. CNS abnormalities associated with precocious puberty include the following: tumors (e.g., astrocytomas, gliomas, germ cell tumors secreting human chorionic gonadotropin [HCG]), hypothalamic hamartomas, acquired CNS injury caused by inflammation, surgery, trauma, radiation therapy, or abscess, or congenital anomalies (e.g. hydrocephalus, arachnoid cysts, suprasellar cysts). These conditions are not

likely in the presence of a normal work-up in this patient. An ovarian neoplasm is unlikely with a normal pelvic ultrasound. c) True precocious puberty would be awarded 10 points An 8 year-old has been diagnosed with precocious puberty due to presence of menarche, Tanner stage III breasts and otherwise normal work-up for brain, adrenal and ovarian abnormalities. What is the most appropriate next step in the management of this patient? a) Depo-Provera b) GnRH agonist c) Danazol d) Estradiol e) Observation True precocious puberty is manifested by premature secretion of GnRH hormone in a pulsatile manner. Once other causes of hormone production are ruled out, treatment would include GnRH agonist to suppress pituitary production of follicular-stimulating hormone and luteinizing hormone. Observation is acceptable if the precocious puberty is within a few months of the routinely expected puberty. The process should be treated if the bone age or puberty is advanced by several years. b) GnRH agonist would be awarded 10 points A 15 year-old adolescent discusses with her doctor that she has not had a menstrual cycle. She is healthy, active in school activities and eats a normal diet. She denies ever being sexually active. On physical examination, she has Tanner stage II breast and pubic hair growth, and average weight and height. Vaginal opening is present and appears normal. What is the most appropriate next step in her management? a) Pelvic ultrasound b) Oral contraceptive pills c) Reassurance d) MRI of Sella Turcica e) Cortisol challenge test Normal age for menarche is between 9 and 17. Since this patient has secondary sexual characteristics and normal anatomy, she should be offered reassurance that she is normal and her menses will probably start soon. c) Reassurance would be awarded 10 points A 4 year-old girl is being evaluated for premature hair growth in the pubic area. She has no breast development and has not had any menstrual bleeding. Laboratory evaluation revealed high DHEA and DHEAS levels and low levels of LH and FSH. Which of the following is the most likely cause of this girls premature adrenarche? a) Idiopathic isosexual precocious puberty b) Congenital adrenal hyperplasia c) Hypothalamic dysfunction d) Pituitary adenoma e) Polycystic ovarian syndrome Congenital adrenal hyperplasia of the 21-hydroxylase type results in the adrenal being unable to produce adequate cortisol as a result of a partial block in the conversion of 17-hydroxyprogesterone to desoxycorticosterone, with the accumulation of adrenal androgens. This leads to precocious adrenarche. Treatment includes steroid replacement. Idiopathic isosexual precocious puberty is GnRH dependent and leads to an appropriate (although early) order of pubertal events. Kallmann Syndrome is a less common cause of hypothalamic dysfunction; the arcuate nucleus does not secrete GnRH. b) Congenital adrenal hyperplasia would be awarded 10 points A 17 year-old is brought to the physician because she has never had a menstrual cycle. She has normal breast and pubic hair development. Physical examination reveals a small vaginal opening with a blind pouch. Pelvic ultrasound reveals normal ovaries, but absence of uterus and cervix. Which of the following is the most appropriate next study in this patient? a) Renal ultrasound b) FSH and LH determination c) Karyotype d) Cortisol level e) Testosterone level Renal anomalies occur in 25-35% of females with Mullerian agenesis. The uterus and cervix are absent, but the

ovaries function normally and, therefore, secondary sexual characteristics are present. You would expect the karyotype in this patient to be 46,XX. a) Renal ultrasound would be awarded 10 points A 13 year-old girl is brought to the physician for increasingly severe abdominal pain. The pain is now a constant low discomfort, but every month she has a week when it is more severe. She has Tanner stage II breast and pubic hair development. On genital examination, there is a bluish mass pushing the labia open. What is the most likely cause of this patients abdominal pain? a) Turners syndrome b) Transverse vaginal septum c) Mullerian agenesis d) Imperforate hymen e) Synechiae of the uterine cavity Lower genital tract malformations occur in 1 in 10,000 females and are most commonly an imperforate hymen where the genital plate canalization is incomplete. The menstrual blood will collect in the vagina and uterus causing pain. Treatment involves surgical correction. When a transverse vaginal septum is present, a normal vaginal opening with a short blind vagina and pelvic mass may be located above the level of the obstruction found on exam. Ashermans syndrome is associated with secondary amenorrhea resulting from intrauterine scarring/synechiae. d) Imperforate hymen would be awarded 10 points A 24-year-old woman comes into the office because she has not had her menses for 6 months. She is in good health and not taking any medications. She is not sexually active. She does well in graduate school, despite her demanding new program. Her height is 5 6 and her weight is 104 pounds. Her blood pressure is 140/80, with other vital signs being stable. Her physical examination, including a pelvic examination, is completely normal. What is the most likely reason for her amenorrhea? a) Ovarian dysfunction b) Undiagnosed diabetes mellitus c) Obstruction of the genital outflow tract at the level of the endocervical canal d) Hypothalamic-Pituitary Dysfunction e) Pregnancy Anorexia nervosa or significant weight loss may cause hypothalamic-pituitary dysfunction that can result in amenorrhea. A lack of the normal pulsatile secretion of gonadotropin releasing hormone (GnRH) leads to a decreased stimulation of the pituitary gland to produce follicle stimulating hormone (FSH) and luteinizing hormone (LH.) This leads to anovulation and amenorrhea. Diabetes mellitus does not primarily result in amenorrhea. While ovarian dysfunction/failure, obstruction of the genital outflow tract and pregnancy cause amenorrhea, they are unlikely in this case. d) Hypothalamic-Pituitary Dysfunction would be awarded 10 points A 23-year-old woman presents to the office because she has not had any menses for 4 months. She has a long history of irregular menstrual cycles since menarche at age 14. She is otherwise in good health and is not taking any medications. She is thin and has chronic anxiety. Her beta-HCG is < 5, and her prolactin and TSH levels are normal. You suspect hypothalamic-pituitary dysfunction as the cause of her amenorrhea. What would be the next best diagnostic test to order? a) Estrogen level b) Transvaginal pelvic ultrasound c) Gonadotropin releasing hormone level d) Follicle stimulating hormone and luteinizing hormone levels e) Progesterone level The causes of hypothalamic-pituitary amenorrhea are functional, (weight loss, obesity, excessive exercise) drugs, (marijuana and tranquilizers) neoplasia, (pituitary adenomas) psychogenic, (chronic anxiety, anorexia nervosa) and certain other chronic medical conditions. In this case, the next step to make a diagnosis is to obtain FSH and LH levels, which would be expected to be in the low range. You already know that her prolactin level is normal, which is consistent with your diagnosis. Prolactin would be elevated with a prolactin-secreting pituitary adenoma. d) Follicle stimulating hormone and luteinizing hormone levels would be awarded 10 points A 23-year-old woman presents to the office because she has not had any menses for 4 months. She has a long history of irregular menstrual cycles since menarche at age 14. She is in good health and is not taking any medications. She is sexually active with her partner of 6 months, and uses condoms for contraception. She is 54 tall and weighs 170 pounds. On exam, she has noticeable hair growth on her upper lip and chin. The rest of her examination including a pelvic exam is normal. Her beta-HCG is < 5, and her prolactin and TSH

levels are normal. In addition to recommending weight loss, what is the most appropriate next step in the management of this patient? a) Initiate treatment with GnRH agonist b) Initiate treatment with steroids c) Initiate treatment with oral contraceptives d) Check Progesterone levels e) Check Cortisol levels Oral contraceptives (OCPs) are the most appropriate treatment for this patient who most likely has the diagnosis of polycystic ovarian syndrome (PCOS.) The constellation of findings support this clinical diagnosis (irregular cycles, obesity, and hirsutism). Because she is using condoms for contraception and is sexually active, OCPs would help regulate her cycles and further provide effective contraception. When she desires pregnancy, however, she will most likely need treatment for ovulation induction due to the anovulatory cycles as the leading cause of her oligomenorrhea. c) Initiate treatment with oral contraceptives would be awarded 10 points A 32-year-old G0 woman presents to the office because she has not had any menses for the last 3 months. She has a long history of irregular cycles, 26 to 45 days apart, for the last two years. She is otherwise in good health and is not taking any medications. She is sexually active with her husband and uses condoms for contraception. She is 5 feet 4 inches tall and weighs 140 pounds. On exam, she has a slightly enlarged, nontender uterus. There are no adnexal masses. What is the most appropriate next step in the management of this patient? a) Perform a pelvic ultrasound b) Check a TSH level c) Check progesterone and estrogen levels d) Perform a urine pregnancy test e) Check FSH and LH levels Pregnancy is the most common cause of amenorrhea. It is important to consider it early in the workup to avoid unnecessary tests, procedures and treatments that may be contraindicated during pregnancy. Although the patient has a history of irregular cycles and is using condoms for contraception, it is important to first rule out pregnancy before initiating further work-up. d) Perform a urine pregnancy test would be awarded 10 points A 33-year-old woman presents to the office because she has not had any menses for the past 12 months. She also reports a recent onset of dyspareunia, causing her to feel anxious about having intercourse. She had menarche at age 15. Her cycles were normal until two years ago when she began skipping menses. She is otherwise in good health. She is 54tall and weighs 130 pounds. Her physical examination is completely normal; TSH and prolactin levels are normal. What is the most likely cause of this patients amenorrhea? a) Psychogemic b) Genital tract outflow obstruction c) Pregnancy d) Premature ovarian failure e) Pituitary adenoma The patients symptom of dyspareunia is likely caused by vaginal dryness, which is associated with estrogen deficiency. Hypergonadotropic amenorrhea is the result of ovarian failure or follicular resistance to gonadotropin stimulation. The history, physical exam and labs make the other possibilities less likely: psychogenic disorder, (no chronic anxiety or anorexia nervosa) outflow obstruction, (previously had periods) pregnancy, (duration too long and normal exam) or a pituitary tumor (normal labs.) d) Premature ovarian failure would be awarded 10 points A 17-year-old woman is brought in by her mother because she has not yet had any menses. She is otherwise in good health, but recently has been experiencing cyclical lower abdominal cramping. She reports never being sexually active. She is 56 tall and weighs 120 pounds. On examination, her breasts are Tanner Stage IV. She has some suprapubic tenderness on abdominal exam. Her pelvic exam reveals normal external genitalia, but there was difficulty inserting a speculum due to patients discomfort. Beta-HCG < 5. What is the most likely diagnosis in this patient? a) Obstruction of the genital outflow tract b) Mllerian agenesis c) Hypothalamic-pituitary dysfunction d) Psychogenic amenorrhea e) Ovarian dysfunction This patients primary amenorrhea, with normal secondary sexual characteristics development and cyclical abdominal

pain, points to an anatomical cause of amenorrhea, which is preventing menstrual bleeding. An imperforate hymen commonly causes this and the treatment is surgical. In Mllerian agenesis, or Mayer-Rokitansky-Kster-Hauser syndrome, there is congential absence of the vagina and usually an absence of the uterus and fallopian tubes. Ovarian function is normal and all the secondary sexual characteristics of puberty occur at the appropriate time. a) Obstruction of the genital outflow tract would be awarded 10 points A 28-year-old G0 female patient presents for an annual examination. She is in good health and not taking any medications. She had a history of normal cycles until 6 months ago, when she stopped having menses after starting an intense exercise regimen. She is 56 tall and weighs 120 pounds. Her examination is completely normal. Her pregnancy test is negative. What is the underlying pathophysiology of the disease process in this patient? a) Psychogenic amenorrhea b) Premature ovarian failure c) Hypothalamic amenorrhea d) Androgen excess e) Hyperthyroidism Amenorrhea associated with exercise falls under the category of hypothalamic amenorrhea, which causes chronic anovulation. Although it may be related to energy requirements, alterations in the hypothalamic-pituitary-ovarian axis have been described in athletic women. The patients history and physical exam make ovarian failure, androgen excess and hyperthyroidism less likely, although a TSH level would still be recommended. c) Hypothalamic amenorrhea would be awarded 10 points A 31-year-old G3P0 female patient comes to the office because she has not had any menses in the last 6 months. She is otherwise in good health and is not taking any medications. She had a miscarriage 7 months ago, which was complicated by an infection and required antibiotics and a dilation and curettage procedure. Her examination is normal. Her laboratory results show a beta-HCG < 5 and normal TSH and Prolactin levels. What is the most likely underlying cause of this patients amenorrhea? a) Chronic endometritis b) Multiple miscarriages c) Hypothalamic pituitary amenorrhea d) Ashermans Syndrome e) Sheehans Syndrome Ashermans syndrome can be caused by curettage or endometritis. The intrauterine synechiae or adhesions result from trauma to the basal layer of the endometrium, which causes amenorrhea. Ovarian failure does not contribute to this disease process. Sheehans syndrome is typically due to severe postpartum hemorrhage leading to pituitary apoplexy. d) Ashermans Syndrome would be awarded 10 points A 23-year-old female college student presents to the office because she has not had any menses for 10 months. She had menarche at age 14 and normal regular menses every 28 days until this year. She is in good health and not taking any medications. She is 54 tall and weighs 130 pounds. Her examination, including a pelvic exam, is normal. Beta-HCG is < 5, and TSH is 5.0 mU/L (normal 0.35-6.7 mU/L). What is the most appropriate next diagnostic test to help determine the cause of amenorrhea in this patient? a) Serum 17-Hydroxyprogesterone levels b) Serum Prolactin levels c) Pelvic ultrasound d) Serum LH and FSH levels e) Brain MRI Measurement of serum prolactin level is part of the initial laboratory assessment for a patient with amenorrhea and no other symptoms or findings on physical exam. A prolactinoma is the most common pituitary tumor causing amenorrhea. Galactorrhea is often present with a hyperprolactinemic cause of anovulation and amenorrhea. A pelvic ultrasound is not typically helpful in a young patient with a normal pelvic examination. A brain MRI might be indicated if prolactin levels return elevated. 17-Hydroxyprogesterone, LH and FSH levels might need to be obtained in the work-up of this patient, if other tests return normal. b) Serum Prolactin levels would be awarded 10 points A 22-year-old G0 female student comes to the office because she has not had any menses since discontinuing her oral contraceptive pills 5 months ago. She has been on the pill for the last 6 years and had normal menses every 28 days while taking them. She is in good health and not taking any medications. She is 54 tall and

weighs 130 pounds. Her examination, including a pelvic exam, is normal. Which of the following history elements would be most useful in determining the cause of amenorrhea in this patient? a) Age at first intercourse b) History of sexually transmitted diseases c) Parity d) History of oligo-ovulatory cycles e) Recent history of weight loss Since most women resume normal menstrual cycles after discontinuing oral contraceptive pills (OCPs,) they are not usually considered the cause of the amenorrhea. A history of irregular cycles prior to pill use may increase the risk of amenorrhea upon discontinuation. This is sometimes referred to as post pill amenorrhea. A complete work-up should be performed to properly find the cause. Although the other history elements are all important components of a complete gynecological history, they are not helpful to find the etiology of amenorrhea in this patient. Significant weight loss might cause amenorrhea; however, this patient still has normal body mass index, which makes it unlikely cause of amenorrhea. d) History of oligo-ovulatory cycles would be awarded 10 points A 24-year-old woman comes into the office because she has not had her menses for 6 months. She is in good health and not taking any medications. She is not sexually active. She does well in graduate school, despite her demanding new program. Her height is 5 6 and her weight is 104 pounds. Her blood pressure is 140/80, with other vital signs being stable. Her physical examination, including a pelvic examination, is completely normal. What is the most likely reason for her amenorrhea? a) Ovarian dysfunction b) Undiagnosed diabetes mellitus c) Obstruction of the genital outflow tract at the level of the endocervical canal d) Hypothalamic-Pituitary Dysfunction e) Pregnancy Anorexia nervosa or significant weight loss may cause hypothalamic-pituitary dysfunction that can result in amenorrhea. A lack of the normal pulsatile secretion of gonadotropin releasing hormone (GnRH) leads to a decreased stimulation of the pituitary gland to produce follicle stimulating hormone (FSH) and luteinizing hormone (LH.) This leads to anovulation and amenorrhea. Diabetes mellitus does not primarily result in amenorrhea. While ovarian dysfunction/failure, obstruction of the genital outflow tract and pregnancy cause amenorrhea, they are unlikely in this case. d) Hypothalamic-Pituitary Dysfunction would be awarded 10 points A 23-year-old woman presents to the office because she has not had any menses for 4 months. She has a long history of irregular menstrual cycles since menarche at age 14. She is otherwise in good health and is not taking any medications. She is thin and has chronic anxiety. Her beta-HCG is < 5, and her prolactin and TSH levels are normal. You suspect hypothalamic-pituitary dysfunction as the cause of her amenorrhea. What would be the next best diagnostic test to order? a) Estrogen level b) Transvaginal pelvic ultrasound c) Gonadotropin releasing hormone level d) Follicle stimulating hormone and luteinizing hormone levels e) Progesterone level The causes of hypothalamic-pituitary amenorrhea are functional, (weight loss, obesity, excessive exercise) drugs, (marijuana and tranquilizers) neoplasia, (pituitary adenomas) psychogenic, (chronic anxiety, anorexia nervosa) and certain other chronic medical conditions. In this case, the next step to make a diagnosis is to obtain FSH and LH levels, which would be expected to be in the low range. You already know that her prolactin level is normal, which is consistent with your diagnosis. Prolactin would be elevated with a prolactin-secreting pituitary adenoma. d) Follicle stimulating hormone and luteinizing hormone levels would be awarded 10 points A 23-year-old woman presents to the office because she has not had any menses for 4 months. She has a long history of irregular menstrual cycles since menarche at age 14. She is in good health and is not taking any medications. She is sexually active with her partner of 6 months, and uses condoms for contraception. She is 54 tall and weighs 170 pounds. On exam, she has noticeable hair growth on her upper lip and chin. The rest of her examination including a pelvic exam is normal. Her beta-HCG is < 5, and her prolactin and TSH levels are normal. In addition to recommending weight loss, what is the most appropriate next step in the management of this patient? a) Initiate treatment with GnRH agonist b) Initiate treatment with steroids c) Initiate treatment with oral contraceptives

d) Check Progesterone levels e) Check Cortisol levels Oral contraceptives (OCPs) are the most appropriate treatment for this patient who most likely has the diagnosis of polycystic ovarian syndrome (PCOS.) The constellation of findings support this clinical diagnosis (irregular cycles, obesity, and hirsutism). Because she is using condoms for contraception and is sexually active, OCPs would help regulate her cycles and further provide effective contraception. When she desires pregnancy, however, she will most likely need treatment for ovulation induction due to the anovulatory cycles as the leading cause of her oligomenorrhea. c) Initiate treatment with oral contraceptives A 32-year-old G0 woman presents to the office because she has not had any menses for the last 3 months. She has a long history of irregular cycles, 26 to 45 days apart, for the last two years. She is otherwise in good health and is not taking any medications. She is sexually active with her husband and uses condoms for contraception. She is 5 feet 4 inches tall and weighs 140 pounds. On exam, she has a slightly enlarged, nontender uterus. There are no adnexal masses. What is the most appropriate next step in the management of this patient? a) Perform a pelvic ultrasound b) Check a TSH level c) Check progesterone and estrogen levels d) Perform a urine pregnancy test e) Check FSH and LH levels Pregnancy is the most common cause of amenorrhea. It is important to consider it early in the workup to avoid unnecessary tests, procedures and treatments that may be contraindicated during pregnancy. Although the patient has a history of irregular cycles and is using condoms for contraception, it is important to first rule out pregnancy before initiating further work-up. d) Perform a urine pregnancy test A 33-year-old woman presents to the office because she has not had any menses for the past 12 months. She also reports a recent onset of dyspareunia, causing her to feel anxious about having intercourse. She had menarche at age 15. Her cycles were normal until two years ago when she began skipping menses. She is otherwise in good health. She is 54tall and weighs 130 pounds. Her physical examination is completely normal; TSH and prolactin levels are normal. What is the most likely cause of this patients amenorrhea? a) Psychogemic b) Genital tract outflow obstruction c) Pregnancy d) Premature ovarian failure e) Pituitary adenoma The patients symptom of dyspareunia is likely caused by vaginal dryness, which is associated with estrogen deficiency. Hypergonadotropic amenorrhea is the result of ovarian failure or follicular resistance to gonadotropin stimulation. The history, physical exam and labs make the other possibilities less likely: psychogenic disorder, (no chronic anxiety or anorexia nervosa) outflow obstruction, (previously had periods) pregnancy, (duration too long and normal exam) or a pituitary tumor (normal labs.) d) Premature ovarian failure A 17-year-old woman is brought in by her mother because she has not yet had any menses. She is otherwise in good health, but recently has been experiencing cyclical lower abdominal cramping. She reports never being sexually active. She is 56 tall and weighs 120 pounds. On examination, her breasts are Tanner Stage IV. She has some suprapubic tenderness on abdominal exam. Her pelvic exam reveals normal external genitalia, but there was difficulty inserting a speculum due to patients discomfort. Beta-HCG < 5. What is the most likely diagnosis in this patient? a) Obstruction of the genital outflow tract b) Mllerian agenesis c) Hypothalamic-pituitary dysfunction d) Psychogenic amenorrhea e) Ovarian dysfunction This patients primary amenorrhea, with normal secondary sexual characteristics development and cyclical abdominal pain, points to an anatomical cause of amenorrhea, which is preventing menstrual bleeding. An imperforate hymen commonly causes this and the treatment is surgical. In Mllerian agenesis, or Mayer-Rokitansky-Kster-Hauser syndrome, there is congential absence of the vagina and usually an absence of the uterus and fallopian tubes. Ovarian function is normal and all the secondary sexual characteristics of puberty occur at the appropriate time. a) Obstruction of the genital outflow tract

A 28-year-old G0 female patient presents for an annual examination. She is in good health and not taking any medications. She had a history of normal cycles until 6 months ago, when she stopped having menses after starting an intense exercise regimen. She is 56 tall and weighs 120 pounds. Her examination is completely normal. Her pregnancy test is negative. What is the underlying pathophysiology of the disease process in this patient? a) Psychogenic amenorrhea b) Premature ovarian failure c) Hypothalamic amenorrhea d) Androgen excess e) Hyperthyroidism Amenorrhea associated with exercise falls under the category of hypothalamic amenorrhea, which causes chronic anovulation. Although it may be related to energy requirements, alterations in the hypothalamic-pituitary-ovarian axis have been described in athletic women. The patients history and physical exam make ovarian failure, androgen excess and hyperthyroidism less likely, although a TSH level would still be recommended. c) Hypothalamic amenorrhea A 31-year-old G3P0 female patient comes to the office because she has not had any menses in the last 6 months. She is otherwise in good health and is not taking any medications. She had a miscarriage 7 months ago, which was complicated by an infection and required antibiotics and a dilation and curettage procedure. Her examination is normal. Her laboratory results show a beta-HCG < 5 and normal TSH and Prolactin levels. What is the most likely underlying cause of this patients amenorrhea? a) Chronic endometritis b) Multiple miscarriages c) Hypothalamic pituitary amenorrhea d) Ashermans Syndrome e) Sheehans Syndrome Ashermans syndrome can be caused by curettage or endometritis. The intrauterine synechiae or adhesions result from trauma to the basal layer of the endometrium, which causes amenorrhea. Ovarian failure does not contribute to this disease process. Sheehans syndrome is typically due to severe postpartum hemorrhage leading to pituitary apoplexy. d) Ashermans Syndrome A 23-year-old female college student presents to the office because she has not had any menses for 10 months. She had menarche at age 14 and normal regular menses every 28 days until this year. She is in good health and not taking any medications. She is 54 tall and weighs 130 pounds. Her examination, including a pelvic exam, is normal. Beta-HCG is < 5, and TSH is 5.0 mU/L (normal 0.35-6.7 mU/L). What is the most appropriate next diagnostic test to help determine the cause of amenorrhea in this patient? a) Serum 17-Hydroxyprogesterone levels b) Serum Prolactin levels c) Pelvic ultrasound d) Serum LH and FSH levels e) Brain MRI Measurement of serum prolactin level is part of the initial laboratory assessment for a patient with amenorrhea and no other symptoms or findings on physical exam. A prolactinoma is the most common pituitary tumor causing amenorrhea. Galactorrhea is often present with a hyperprolactinemic cause of anovulation and amenorrhea. A pelvic ultrasound is not typically helpful in a young patient with a normal pelvic examination. A brain MRI might be indicated if prolactin levels return elevated. 17-Hydroxyprogesterone, LH and FSH levels might need to be obtained in the work-up of this patient, if other tests return normal. b) Serum Prolactin levels A 22-year-old G0 female student comes to the office because she has not had any menses since discontinuing her oral contraceptive pills 5 months ago. She has been on the pill for the last 6 years and had normal menses every 28 days while taking them. She is in good health and not taking any medications. She is 54 tall and weighs 130 pounds. Her examination, including a pelvic exam, is normal. Which of the following history elements would be most useful in determining the cause of amenorrhea in this patient? a) Age at first intercourse b) History of sexually transmitted diseases c) Parity

d) History of oligo-ovulatory cycles e) Recent history of weight loss Since most women resume normal menstrual cycles after discontinuing oral contraceptive pills (OCPs,) they are not usually considered the cause of the amenorrhea. A history of irregular cycles prior to pill use may increase the risk of amenorrhea upon discontinuation. This is sometimes referred to as post pill amenorrhea. A complete work-up should be performed to properly find the cause. Although the other history elements are all important components of a complete gynecological history, they are not helpful to find the etiology of amenorrhea in this patient. Significant weight loss might cause amenorrhea; however, this patient still has normal body mass index, which makes it unlikely cause of amenorrhea. d) History of oligo-ovulatory cycles A 32 year-old G0 woman presents with irregular menses occurring every 6-8 weeks for the past 8 months. The bleeding alternates between light and heavy. Her irregular menses were treated successfully with Medroxyprogesterone Acetate (MPA), 10 mg every day, taken for 10 days each month. By which mechanism does the MPA control her periods? a) Stimulates rapid endometrial growth and regeneration of glandular stumps b) Converts endometrium from proliferative to secretory c) Promotes release of Prostaglandin F2 d) Regenerates functional layer of the endometrium e) Decreases luteal phase inhibin production Patients with anovulatory bleeding have predominantly proliferative endometrium from unopposed stimulation by estrogen. Progestins inhibit further endometrial growth, converting the proliferative to secretory endometrium. Withdrawal of the progestin then mimics the effect of the involution of the corpus luteum, creating a normal sloughing of the endometrium. Stimulation of rapid endometrial growth, conversion of proliferative to secretory endometrium, and regeneration of the functional layer describe effects of estrogen on the endometrium. Inhibin is increased in the luteal phase. b) Converts endometrium from proliferative to secretory A 41 year-old G3P3 woman reports heavy menstrual periods occurring every 26 days lasting 8 days. The periods have been increasingly heavy over the last three months. She reports soaking through pads and tampons every 2 hours. She has a history of three uncomplicated spontaneous vaginal deliveries and a tubal ligation following the birth of her last child. On pelvic examination, the cervix appears normal and the uterus is normal in size. Which of the following tests or procedures would be most useful in further evaluation of this patients complaint? a) Follicle stimulating hormone level b) Prolactin level c) Coagulation studies d) Pelvic ultrasound e) CT of the pelvis A pelvic ultrasound would image the endometrium and rule in or rule out uterine, and possibly endometrial, polyps. In the absence of menopausal symptoms, FSH is unlikely to be helpful. The patient is unlikely to have a coagulation disorder, as she has had three spontaneous vaginal deliveries without postpartum hemorrhage. CT is not as accurate a modality for evaluating the uterus as ultrasound. Hyperprolactinemia is found with prolactin-secreting adenomas associated with amenorrhea. d) Pelvic ultrasound A 14-year old G0 adolescent reports menarche six months ago, with increasingly heavy menstrual flow causing her to miss several days of school. Three months ago, her pediatrician started her on oral contraceptives to control her menstrual periods, but she continues to bleed heavily. Her previous medical history is unremarkable. The patient has a normal body habitus for her age. Appropriate breast and pubic hair development is present. Her hemoglobin is 9.1 mg/dl, hematocrit 27.8%, urine pregnancy test negative. Which of the following etiologies for menorrhagia is most likely the cause of her symptoms? a) Uterine leiomyoma b) Thyroid disorder c) Coagulation disorder d) Endometrial hyperplasia e) Chronic Endometritis Disorders of clotting may present with menstrual symptoms in young women, with Von Willeberand disease being most common. Leiomyomas typically present in women in their 30s and 40s. Endometrial hyperplasia can occur in younger anovulatory patients, but the short duration of this patients symptoms makes this less likely. She does not have any signs of infection.

c) Coagulation disorder A 42 year-old G2P2 woman presents with menorrhagia. Her menses occur every 28 days. She noted that the bleeding became worse after the birth of her last child and it has increased in severity over the last two years. She is currently taking combination oral contraceptives, which were started six months ago by her family physician. On pelvic examination, the uterus is normal in size and no adnexal masses are noted. Her hemoglobin is 11.9 mg/dl, hematocrit 35.1%. Pelvic sonography reveals a normal appearing uterus and a normal appearing endometrial echo. The ovaries appear normal without adnexal masses. Endometrial biopsy reveals a secretory endometrium. The patient is unhappy with the current therapy and desires definitive treatment. Which of the following would be the best long-term treatment? a) Endometrial ablation b) Dilatation and Curettage c) Laparoscopic myomectomy d) Leuprolide acetate e) Cyclic Progestins Endometrial ablation is an effective surgical procedure for dysfunctional uterine bleeding (DUB). Dilation and curettage is not effective as a therapy for DUB in the long-term. Leuprolide should not be used for more than 6 months because of the risk of osteoporosis. Cyclic progestins are unlikely to work any better than oral contraceptives with ovulatory DUB. Pelvic exam and ultrasound did not reveal any fibroids, making a myomectomy unnecessary. a) Endometrial ablation A 35 year-old G0 woman presents with irregular menstrual periods occurring every 6-12 weeks with occasional inter-menstrual bleeding. Currently, she has been bleeding daily for the last 4 weeks. She reports that her periods have always been irregular, but have become more so with heavier flow and cramping in the last year. She is sexually active with one partner. On physical exam, she is morbidly obese with no abnormalities detected on pelvic exam. Which of the following is the most appropriate next step in the management of this patient? a) Luteinizing hormone level (LH) b) Pelvic CT c) Testosterone level d) Follicle stimulating hormone level (FSH) e) Endometrial biopsy Endometrial biopsy should be performed to rule out endometrial hyperplasia or carcinoma given the history of irregular bleeding, coupled with the increased risk of these diagnoses in morbidly obese patients. A pelvic CT would not evaluate for the presence of endometrial neoplasia. LH and FSH levels would not aid in the diagnostic workup and testosterone levels would not be useful, unless signs of hirsutism or virilization are present. e) Endometrial biopsy A 34 year-old G2P2 woman presents with inter-menstrual bleeding for one year. The bleeding typically occurs 2 weeks after her menses and last 2-3 days. The symptoms began 1 year ago and the bleeding has not changed recently. She is currently taking oral contraceptives. On pelvic examination, the cervix appears normal and the uterus is normal in size and shape. Her urine pregnancy test is negative; an endometrial biopsy is negative for neoplasia. Which of the following tests or procedures would be indicated for further work-up? a) Prolactin level b) Progesterone level c) Hysterosalpingogram (HSG) d) Pelvic ultrasound e) Colposcopy Intermenstrual bleeding is frequently caused by structural abnormalities of the endometrial cavity, such as myomas, polyps or malignancy. An ultrasound would be helpful as the next step in diagnosis. Although an HSG might reveal structural abnormalities, it is too invasive for a next step. A colposcopy would not be helpful in the diagnosis, nor would obtaining a Prolactin level, as it would be indicated for the evaluation of anovulatory bleeding. Progesterone levels are not helpful in a patient on oral contraceptives. d) Pelvic ultrasound A 36 year-old G0 woman presents due to increasing facial hair growth and irregular menstrual cycles. She has gained 40 pounds over the last 3 years. Her symptoms began 3 years ago and have gradually worsened. She has never been pregnant and is not currently on any medications. On physical exam, she is overweight with

dark hair growth at the sideburns and upper lip. The pelvic exam is normal. Which of the following would you expect to find in this patient? a) Decreased luteinizing hormone levels b) Elevated free testosterone c) Decreased prolactin level d) Increased ovarian estrogen production e) Elevated 17-hydroxyprogesterone This patient likely has polycystic ovarian syndrome (PCOS). PCOS patients have testosterone levels at the upper limits of normal or slightly increased. Free testosterone (biologically active) is elevated often because sex hormone binding globulin is decreased by elevated androgens. LH is increased in response to increased circulating estrogens fed by an elevation of ovarian androgen production. Insulin resistance and chronic anovulation are hallmarks of PCOS. Prolactin levels may be elevated in amenorrhea but are not elevated in patients with PCOS. b) Elevated free testosterone An 18 year-old woman comes to the office due to vaginal spotting for the last two weeks. Her menstrual periods were regular until last month, occurring every 28-32 days. Menarche was at age 13. She started oral contraceptives 3 months ago. On pelvic examination, the uterus is normal in size, slightly tender with a mass palpable in the right adnexal region. No adnexal tenderness is noted. Which of the following tests is the most appropriate next step in the management of this patient? a) Endometrial biopsy b) Coagulation studies c) Pelvic sonography d) Abdominal CT scan e) Urine pregnancy test It is vitally important to rule out pregnancy in the evaluation of abnormal uterine bleeding. Although a bleeding disorder could be a possibility, it is less likely with her previous history of normal cycles. Sonography could be considered as a next step if the pregnancy test is negative in order to evaluate the adnexal finding. Abdominal CT would not be performed in this patient unless advanced adnexal pathology was found on pelvic sonography. Endometrial biopsy would rarely be indicated in a teen with abnormal bleeding, unless morbidly obese and anovulatory. e) Urine pregnancy test A 45 year-old G2P2 woman comes to the office because of heavy and irregular menstrual periods. The heavy periods started three years ago and have gradually worsened in amount of flow over time. The periods are interfering with her daily activities. The patient has had two spontaneous vaginal deliveries, followed by a tubal ligation 3 years ago. On pelvic examination, the cervix appears normal and the uterus is normal in size without adnexal masses or tenderness. A urine pregnancy test is negative. TSH and prolactin levels are normal. Hemoglobin is 12.5 mg/dl. On pelvic sonography, she has a normal size uterus and a 2 cm simple cyst on the right ovary. Endometrial biopsy is consistent with a secretory endometrium; no neoplasia is found. What is the most likely diagnosis in this patient? a) Polycystic ovarian syndrome b) Mid-cycle bleeding c) Dysfunctional uterine bleeding d) Benign cystic teratoma e) Ovarian cancer Dysfunctional uterine bleeding is defined as irregular or increased menstrual bleeding without identified etiology. This patient had a complete workup, including TSH, Prolactin, pelvic ultrasound and endometrial biopsy, which were all normal. Mid-cycle bleeding at the time of ovulation is due to the drop in estrogen. Ovarian teratomas are unlikely to present with abnormal menses. They typically present with abdominal or pelvic pain which may be associated with torsion. The 2 cm cyst is a functional cyst and is a common finding in ovulatory patients. c) Dysfunctional uterine bleeding A 35 year-old G2P2 woman comes to the office due to heavy menstrual periods. The heavy periods started three years ago and have gradually worsened in amount of flow and duration. The periods are now interfering with her daily activities. The patient had two spontaneous vaginal deliveries. She smokes 1 pack of cigarettes per day. On pelvic examination, the cervix appears normal and the uterus is normal in size, without adnexal masses or tenderness. A urine pregnancy test is negative. TSH and prolactin levels are normal. Hemoglobin is 12.5 mg/dl. On pelvic sonography, a 2 cm submucosal leiomyoma is noted. An endometrial biopsy is consistent with a secretory endometrium; no neoplasia is found. Which of the following would be the best therapeutic option for this patient if she desires to have another child?

a) Hysteroscopy with myoma resection b) Laparoscopic myomectomy c) Endometrial ablation d) Oral contraceptives e) Dilatation and Curettage Hysteroscopic myomectomy preserves the uterus, while removing the pathology causing the patients symptoms. A laparoscopic approach is not indicated as the myoma is submucosal and not accessible using a laparoscopic approach. Endometrial ablation destroys the endometrium and can create Ashermans syndrome, thus it is reserved for patients who have completed childbearing. Dilation and curettage is unlikely to remove the myoma and is a blind procedure (carried out without direct visualization). Oral contraceptives would typically help with heavy menses, but are contraindicated in this patient, who is over 35 and smokes. a) Hysteroscopy with myoma resection A 19 year-old G0 presents with severe menstrual pain which causes her to miss school. She takes 600 mg of ibuprofen every 4-6 hours to control the pain, but this does not relieve the discomfort. She is sexually active, with one present partner (two lifetime partners) and uses condoms for contraception. Examination is normal. What is the most appropriate next step in the management of this patient? a) Change NSAID to a COX-2 inhibitor b) GnRH agonist c) Oral contraceptives d) Continuous medroxyprogesterone e) Laparoscopy Dysmenorrhea or painful menstrual cramps is often incapacitating. Oral contraceptives will not only relieve primary dysmenorrhea, but also provide more reliable contraception. COX-2 inhibitors have targeted action but have significant side effects, and are no longer routinely prescribed. Continuous oral Medroxyprogesterone may be effective, but will not provide contraception. Depo-Provera would be a better choice. GnRH agonists are too expensive and have too high a side effect profile to be used for this purpose. c) Oral contraceptives A 15 year-old G0 presents with severe menstrual pain for the past 12 months. The pain is severe enough for her to miss school. The pain is not relieved with ibuprofen 600 mg every 4 hours. She is not sexually active and the workup reveals no pathology. The most appropriate next step in the management of this patient is to begin combination oral contraceptives. How do oral contraceptives relieve primary dysmenorrhea? a) Creating endometrial atrophy b) Decreasing inflammation c) Increasing prolactin levels d) Decreasing inhibin levels e) Thickening cervical mucous The progestin in oral contraceptives creates endometrial atrophy. Since prostaglandins are produced in the endometrium, there would be less produced. Dysmenorrhea should be improved. a) Creating endometrial atrophy A 21 year-old G0 presents with severe menstrual pain. She takes 600 mg of ibuprofen every 4-6 hours to control the pain, but this does not relieve the discomfort. She is sexually active with one present partner and has four lifetime partners. She uses condoms for contraception. Past medical history is unremarkable, except for breast cancer in her fathers sister. Examination is normal. In addition to a Pap smear, what is the most appropriate additional test needed for this patient? a) Baseline mammography b) Chlamydia testing c) HIV testing d) HPV DNA typing e) Lipid profile The U.S. Preventive Services Task Force recommends chlamydia and gonorrhea screening for all sexually active patients, age 25 and younger. Since pelvic inflammatory disease is a cause of secondary dysmenorrhea, it needs to be evaluated as a potential cause of her symptoms. Although HPV screening is becoming more common, it can be used as an adjunct to cytology in primary screening in women 30 years or older, and is not indicated in a 21 year-old unless her Pap smear shows ASCUS. Lipid screening is not necessary in a 21 year-old otherwise healthy woman with no risk factors. b) Chlamydia testing

A 19 year-old G0 woman presents with severe menstrual pain that causes her to miss school. She takes 600 mg of ibuprofen every 4-6 hours to control the pain, but this does not relieve the discomfort. You started oral contraceptives, but her symptoms persisted. She also tried Depo-Provera for 3 months without much improvement. She still has menstrual pain and continues to miss some classes. What is the most appropriate next step in the management? a) Transdermal narcotic for pain relief b) Diagnostic laparoscopy c) Presacral neurectomy d) Prescribe a selective serotonin reuptake inhibitor e) Prescribe GnRH agonists Laparoscopy is recommended to confirm the diagnosis of endometriosis and exclude other causes of secondary dysmenorrhea. Some authors suggest that a course of GnRH agonists are appropriate, with laparoscopy reserved for those women who have pain during or after completion of a 3-month course. b) Diagnostic laparoscopy A 23 year-old G0 woman with severe dysmenorrhea that is unresponsive to non-steroidal anti-inflammatory agents and oral contraceptives is taken to the operating room for a laparoscopy. Blue-black powder burn lesions are seen in the pelvis. A biopsy is performed and sent to pathology. Which of the following pathologic lesions would you expect to see in this patient? a) Blue-domed cysts greater than 3 mm b) Decidual effect in the endometrium c) Endometrial glands or stroma and hemosiderin-laden macrophages d) Invasion of endometrial glands into the myometrium e) Well-circumscribed, non-encapsulated myometrium The lesions described are classic for endometriosis. One would therefore expect to see endometrial glands/stroma with hemosiderin-laden macrophages. c) Endometrial glands or stroma and hemosiderin-laden macrophages A 42 year-old G4P4 woman presents with a history of progressively worsening severe menstrual pain. Menses are regular, but she complains of very heavy flow requiring both a menstrual pad and tampon with frequent bleeds through this protection on heavy days. She takes Oxycodone that her husband used for back pain to relieve her dysmenorrhea. She had a tubal ligation four years ago. Pelvic examination shows an enlarged, soft, boggy uterus. No masses are palpated. Pregnancy test is negative, hemoglobin 9.8 and hematocrit 28.3%. What is the most likely diagnosis? a) Adenomyosis b) Endometrial carcinoma c) Endometriosis d) Primary dysmenorrhea e) Endometrial hyperplasia This is a typical presentation of adenomyosis (presence of endometrial glands and supporting tissues in the muscle of the uterus). The gland tissue grows during the menstrual cycle and, at menses, tries to slough, but cannot escape the uterine muscle and flow out of the cervix as part of normal menses. This trapping of the blood and tissue causes uterine pain in the form of monthly menstrual cramps. Endometrial hyperplasia and carcinoma are less likely in a woman with regular menses. Endometriosis would most likely have presented earlier in life and would not explain the enlarged uterus. a) Adenomyosis A 42 year-old G4P4 woman presents for management of suspected adenomyosis. She had a tubal ligation four years ago. A pelvic examination shows an enlarged, soft, boggy uterus. A pregnancy test is negative and she is mildly anemic. An ultrasound shows an enlarged uterus with no fibroids. The patient desires definitive treatment for this condition. What is the most appropriate next step in her management? a) Continuous estrogen/progestin therapy b) Endometrial ablation c) GNRH agonists d) Hysterectomy e) Insertion of a levonorgestrel containing intrauterine system Hysterectomy is nearly 80% effective in eliminating pain and abnormal bleeding, if she is willing to undergo surgery. Gonadotropin releasing agents are the first choice for medical therapy for the pain, but the problem is that the adenomyosis seems to recur after discontinuing the therapy. Endometrial ablation and insertion of a levonorgestrelcontaining intrauterine system are options in women who decline hysterectomy or desire to maintain fertility. For

abnormal bleeding problems and desire for uterine conservation, a progesterone intrauterine contraceptive device can also be used to improve irregular bleeding. Hysteroscopic endometrial ablation can be a treatment for adenomyosis. d) Hysterectomy A 43 year-old G4P4 woman presents with a history of progressively worsening, severe menstrual pain. Her workup is consistent with adenomyosis, and she selects hysterectomy for definitive treatment. What is the pathologic lesion to confirm the diagnosis? a) Blue-domed cysts greater than 3 mm. b) Decidual effect in the endometrium c) Endometrial glands/stroma and hemosiderin-laden macrophages d) Invasion of endometrial glands into the myometrium e) Well-circumscribed, non-encapsulated myometrium Invasion of endometrial glands into the myometrium is the pathological hallmark of adenomyosis. d) Invasion of endometrial glands into the myometrium A 41 year-old G2P2 woman presents with menstrual pain, menorrhagia, irregular periods and intermenstrual bleeding. She describes the pain as pressure and cramps. Ibuprofen improves the pain, but does not entirely eliminate the discomfort. Pelvic examination reveals a 14-week size uterus with irregular masses within the uterus. Pelvic ultrasound confirms the diagnosis of fibroids. What is the most appropriate next step in the management of this patient? a) CA125 assay b) CT scan of the pelvis c) Endometrial biopsy d) GnRH agonists e) Hysterectomy This patient has classic symptoms of leiomyomata, including menorrhagia. An endometrial biopsy should be performed on all women over age 40 with irregular bleeding to rule out endometrial carcinoma. The CA125 assay measures the level of CA125 in the blood and is increased in some types of cancer, including ovarian cancer or other conditions. This non-specific marker is not indicated in this patient. A CT scan of the pelvis is also not indicated. A simple pelvic ultrasound could be used to help confirm the clinical diagnosis. GnRH agonist and hysterectomy are not used until the diagnosis of leiomyomata uteri is confirmed. Her desire for future fertility should be discussed. c) Endometrial biopsy A 42 year-old G2P2 woman undergoes a hysterectomy for definitive treatment of her dysmenorrhea and large uterine fibroids. The uterus is sent to pathology. Which of the following would confirm the diagnosis of fibroids? a) Blue-domed cysts greater than 3 mm b) Decidual effect in the endometrium c) Endometrial glands/stroma and hemosiderin-laden macrophages d) Invasion of endometrial glands into the myometrium e) Well-circumscribed, non-encapsulated myometrium Well-circumscribed, non-encapsulated myometrium confirms the diagnosis of fibroids. Leiomyosarcomas will have > than 10 mitotic figures per high power field. e) Well-circumscribed, non-encapsulated myometrium A 53 year-old G2P2 woman comes to your office complaining of 6 months of worsening hot flashes, vaginal dryness, night sweats and sleep disturbances. Her last normal menstrual period was 6 months ago and she has been experiencing intermittent small amounts of vaginal bleeding. Her medical history is significant for hypertension, which is well-controlled by a calcium-channel blocker, adult onset diabetes, for which she takes Metformin, and hyperthyroidism, for which she takes Propylthiouracil. The patient is 57 tall and weighs 140 pounds. Blood pressure is 120/70. Physical examination is unremarkable. Which of the following medical conditions in this patient is a contraindication to treatment of menopausal symptoms with hormone therapy? a) Vaginal bleeding b) Hypertension c) Diabetes d) Osteoporosis e) Hyperthyroidism The principal symptom of endometrial cancer is abnormal vaginal bleeding. Although the patients worsening symptoms makes treatment an important consideration, the specific organic cause(s) of abnormal bleeding must be ruled out prior to initiating therapy. A tissue diagnosis consistent with normal endometrium or a pelvic ultrasound with

an endometrial stripe of <4 mm ought to be documented. In addition, risks and benefits of hormone replacement therapy must be discussed with this patient at length prior to beginning treatment. a) Vaginal bleeding A 47 year-old G2P2 woman comes to your office because she has skipped her menstrual period for the past 3 months. She denies any menopausal symptoms. Review of symptoms and physical exam are unremarkable. Quantitative BHCG <5 mIU/ml, TSH= 1.2 mIU/L (normal). What is the most likely diagnosis in this patient? a) Hypothyroidism b) Early pregnancy c) Perimenopause d) Premature ovarian failure e) Autoimmune disorder Although there has been a decline in the average age of menarche with the improvement in health and living conditions, the average age of menopause has remained stable. The Massachusetts Womens Health Study reports that the average age of menopause is 51.3. This patient is most likely perimenopausal and will probably have more menstrual periods in the future. She has normal BHCG and TSH. Premature ovarian failure occurs before age 35. Her normal TSH rules out hypothyroidism and she lacks signs and symptoms of other autoimmune disorders. c) Perimenopause A 58 year-old G3P3 woman has been postmenopausal for 5 years and is concerned about osteoporosis. She has declined hormone therapy in the past. Her mother has a history of a hip fracture at age 82. A physical exam is unremarkable. In addition to weight bearing exercise and vitamin D supplementation, what optimal daily calcium intake should she take? a) 500 mg b) 750 mg c) 1000 mg d) 1200 mg e) 2000 mg Calcium absorption decreases with age because of a decrease in biologically active vitamin D. A positive calcium balance is necessary to prevent osteoporosis. Calcium supplementation reduces bone loss and decreases fractures in individuals with low dietary intakes. In order to remain in zero calcium balance, postmenopausal women require a total of 1200 to 1500 mg of elemental calcium per day. d) 1200 mg A 58 year-old G3P1 woman presents to your office for her annual exam. She became menopausal at age 54. Her past medical history is significant for angina. She experienced a Colles fracture 14 months ago when she tripped and fell while running after her grandson. She has not had any surgeries. She takes no medications and has no known drug allergies. She smokes 10 cigarettes a day and drinks a glass of red wine at dinner. Her father was diagnosed with colon cancer at the age of 72. Physical exam revealed a BP =120/68, P=64, BMI= 22. Her heart, lung, breast and abdominal exams were normal. Pelvic exam was consistent with vaginal atrophy and a small uterus. There was no adnexal tenderness and no masses were palpated. What is the next step in the management plan for this patient? a) Begin screening for osteoporosis at age 65 b) DEXA scan now and treat if T score is < -1.5 c) Begin hormone replacement therapy d) Begin treatment with bisphosphonates e) Test for the presence of biochemical bone markers in the blood This patient has many of the major risk factors for osteoporosis including history of fracture as an adult, low body weight and being a current smoker. General recommendations for the prevention of osteoporosis include eating a balanced diet that includes adequate intake of calcium and vitamin D, regular physical activity, avoidance of heavy alcohol consumption, and smoking cessation. DEXA is the test of choice for measuring bone mineral density (BMD). Bone markers are used in research but are not yet a reliable predictor of BMD. Patients who already have had an osteoporotic fracture may be treated on this basis alone, although a DEXA scan may be useful for other reasons, such as ruling out a pathologic fracture from metastatic disease. Hormone replacement therapy is not recommended long term for disease prevention especially in patients with cardiovascular disease. Prior to beginning treatment with bisphosphonates, a BMD should be documented and repeated at two -year intervals to monitor treatment. d) Begin treatment with bisphosphonates A 54 year-old G2P2 presents to your office for an annual visit. Her last menstrual period was 8 months ago. She complains of severe vasomotor symptoms, vaginal dryness, and dyspareunia and she desires treatment

for her symptoms. She has otherwise been in good health. She has no significant past medical or past surgical history. Her family history is significant for a mother who has severe osteoporosis at the age of 75 and a grandmother who died of breast cancer at the age of 79. She does not report any smoking, alcohol or drug use. On physical exam her BP is 130/78, Pulse 84, BMI is 26. The remainder of her exam is within normal limits except for severe vaginal atrophy noted on the pelvic examination. The best recommendation for this patient would include which of the following? a) Lowest effective dose of combination hormone replacement therapy for the shortest duration possible b) Long term hormone replacement therapy to treat her vasomotor symptoms and prevention of osteoporosis c) Testosterone cream d) Progesterone cream e) Biosphophonates The ACOG 2004 report on hormone replacement therapy considers hormone replacement therapy (HRT) the most effective treatment for severe menopausal symptoms that include hot flashes, night sweats and vaginal dryness. The physician should counsel the woman about the risks and benefits before initiating treatment. ACOG recommends the smallest effective dose for the shortest possible time and annual reviews of the decision to take hormones. HRT should not be used to prevent cardiovascular disease due to the slight increase in risk of breast cancer, myocardial infarction, cerebrovascular accident, and thromoboembolic events. A woman with an intact uterus should not use estrogen-only therapy because of the increased risk of endometrial cancer. In addition to the same risks as FDA approved treatments, bioidentical hormones such as testosterone and progesterone cream may have additional associated risks. Bisphosphonates are used to treat osteoporosis and will not relieve her symptoms. a) Lowest effective dose of combination hormone replacement therapy for the shortest duration possible A 52 year-old G3P3 woman presents to your office with severe hot flashes and vaginal dryness for 6 months. Her last menstrual period was 15 months ago. After discussing the risks and benefits of hormone therapy with this patient, she decides to begin treatment. This patient is most likely to stop hormone therapy secondary to what side effect? a) Vaginal bleeding b) Development of breast cancer c) Hirsutism d) Nausea e) Relief of menopausal symptoms Most irregular bleeding due to initiation of hormone therapy occurs in the first 6 months. It has been cited as the most common reason as to why women stop hormone therapy. Women who are amenorrheic for some period of time are often disturbed by the resumption of any vaginal bleeding/spotting and find it intolerable. a) Vaginal bleeding 49 year-old G2P2 woman s/p hysterectomy at age 45 for fibroids presents to your office complaining of severe vasomotor symptoms for 3 months. Hot flashes are affecting her quality of life and she would like to discuss options for treatment. What treatment option for hot flushes associated with menopause do you recommend as the most effective? a) Lifestyle modifications such as dressing in layers b) Estrogen c) Selective estrogen receptor modulator (SERMs) d) Selective serotonin reuptake inhibitors (SSRIs) e) Treatment with Phytoestrogen (soy) Except for estrogen receptor modulator therapy, all of the above treatment options will improve hot flash symptoms. Treatment with estrogen is most effective, and the current recommendation is for the lowest dose for the shortest duration of time. Hot flashes will resolve completely in 90% of patients receiving this therapy. Raloxifene, a selective estrogen receptor modulator, may actually cause hot flashes to worsen in a patient who has not stopped having these symptoms completely. SSRI antidepressants, some anti-seizure medications and alternative treatments, such as soy products and herbs, have not been shown to be as effective as estrogen. b) Estrogen A 49 year-old G1P1 woman comes to your office for menopause counseling. She has been experiencing severe sleep disturbances and night sweats for the past 4 months. She would like to begin hormone therapy, but is concerned because she has elevated cholesterol levels for which she takes medication. You explain to her that hormone therapy has the following effect on a lipid/cholesterol profile: a) Both LDL and HDL levels increase b) Both HDL and LDL levels decrease c) HDL and LDL levels are unaffected

d) HDL levels increase and LDL levels are unaffected e) HDL levels increase and LDL levels decrease Recent data have confirmed the overall positive effects of hormone therapy on serum lipid profiles. The most important lipid effects of postmenopausal hormone treatment are the reduction in LDL cholesterol and the increase in HDL cholesterol. Estrogen increases triglycerides and increases LDL catabolism, as well as lipoprotein receptor numbers and activity, therefore causing decreased LDL levels. Hormones inhibit hepatic lipase activity, which prevents conversion of HDL2 to HDL3, thus increasing HDL levels. Hormone therapy is not currently recommended for the primary prevention of heart disease. e) HDL levels increase and LDL levels decrease A 59 year-old G0 woman comes to your office for an annual exam. The patient has been postmenopausal for 5 years. She is complaining of vaginal dryness and pain with intercourse for the past 3 years. If you were to check, what would be this patients circulating estradiol level? a) 10-20 pg/ml b) 50-60 pg/ml c) 100-150 pg/ml d) 200-250 pg/ml e) 400-450 pg/ml The circulating estradiol level after menopause is approximately 10-20 pg/ml. This is derived from the peripheral conversion of estrone, which is mainly a result of the peripheral conversion of androstenedione. a) 10-20 pg/ml A 54 year-old G4P4 woman who has been menopausal for 4 years comes to you for an annual exam. She is in good health, eats a balanced diet, exercises regularly, and has never had any menopausal symptoms and wants to know why. You explain to her that some untreated postmenopausal women will have circulating estrogen levels that are adequate to prevent them from experiencing the symptoms of menopause. What is the most likely source of these circulating estrogens? a) Exogenous dietary intake b) Limited adrenal estrogen secretion c) Lingering ovarian estrogen d) Aromatization of circulating androgens e) Continued low-level ovarian production Estrogen production by the ovaries does not continue beyond the menopause. Estrogen levels in postmenopausal women can be significant, due to the extraglandular conversion of androstenedione and testosterone to estrogen. This conversion occurs in peripheral fat cells and thus, body weight has been directly correlated with circulating levels of estrone and estradiol. d) Aromatization of circulating androgens

Вам также может понравиться